Sunteți pe pagina 1din 64

CUPRINS

Cuvânt înainte din partea colectivului redacţional _________________ 1


Membrii Filialei Mehedinţi a S.S.M.R. ____________________________ 2

Concursul Interjudeţean de Matematică “Petre Sergescu”


Subiecte_________________________________________________________ 3
Premianţi ________________________________________________________ 6
Simpozionul de referate _____________________________________________ 7

Note matematice
Pregatire pentru olimpiade şi concursuri _________________________ 10
Probleme pentru Olimpiada ______________________________________ 11
Asupra Numerelor Algebrice _____________________________________ 14

Teme pentru grupele de performanţă


Clasa a V-a
Operaţii cu numere naturale binare ______________________________ 16
Clasa a V-a
Selectie de probleme __________________________________________ 18
Clasa a VII-a
Operaţii cu numere iraţionale _____________________________________ 20
Clasa a VIII-a
Principiul Invariantului __________________________________________ 23
Clasa a IX-a
Invarianţi ________________________________________________________ 29
Clasa a X-a
Câteva probleme generate de exponenţiala de bază 2 ______________ 27
Clasa a XII-a
Centralizatorul unui element într-un grup. Centrul unui grup.
Teorema lui Lagrange. Indicele unui subgrup într-un grup _________ 29
Clasa a XII-a
Asupra elementelor simetrizabile dintr-un monoid _________________ 32
Clasa a XII-a
Extinderi de inele şi corpuri _______________________________________ 34

Probleme propuse ________________________________________________ 36


Subiecte Olimpiada de matematică etapa locală 2008 _____________ 45
Teste de verificare – admitere clasa a V-a _________________________ 48

Premianţii concursurilor din 2007 ______________________________ 52


Rubrica rezolvitorilor ________________________________________________ 60

-0-
EDITORIAL SSM
H
Cuvânt înainte din partea colectivului
redacţional

Daca aceste randuri au ajuns in fata dumneavoastra inseamna ca am


reusit si in acest an sa realizam Revista Mehedinteana de
Matematica.Publicatia noastra ajunge astfel la nr.8 si va reaminteste ca
scopul ei,izvorat din Statutul Societatii de Stiinte Matematice din Romania
este de a contribui la ridicarea nivelului de pregatire matematica a elevilor si
profesorilor mehedinteni si nu numai.
In paginile revistei veti gasi reflectate o parte din activitatile Filialei
Mehedinti a SSMR, lista membrilor filialei noastre la 31.12.2007(revista se
concepe in perioada septembrie-noiembrie 2008 si deci asteptam pana la
sfarsitul anului ca toti iubitorii de matematica sa ni se alature in continuare
prin plata cotizatiei pe 2008),asa cum ne-am obisnuit deja Teme pentru
grupele de performanta,problemele Editiei a IV-a a Concursului
Interjudetean Petre Sergescu (cu bucurie semnalam ca sora noastra mai
mare gazeta Matematica publica in Nr.9/2008 un material care reflecta
concursul nostrum),probleme propuse ,lista rezolvitorilor care au trimis
probleme rezolvate prin intermediul colegilor nostri entuziasti de la scolile
mehedintene,si nu in ultimul rand o reflectare speram noi cat mai fidela a
rezultatelor obtinute de elevii nostri la concursurile interjudetene,nationale
si internationale la care au participat.
Problemele invatamantului sunt multe ,dar noi cei care lucram cel
putin pe teritoriul privilegiat al matematicii nu putem accepta in niciun caz
concluzii semidocte afisate pe la diverse posturi de televiziune ca profesorii
romani au o slaba pregatire. Credem ca si cu acesta revista putem arata
oricui pe mapamond ca pregatirea dascalilor nostri este profunda si ca
rezultatele lor sunt meritorii. Noi, un grup mare de profesori de matematica,
suntem convinsi ca elevii nostrii au valoare,sunt bine instruiti si pot
concura oricand cu colegii lor din restul lumii. Noi credem ca pregatirea
matematica a tuturor elevilor si nu numai a olimpicilor este extrem de utila
pentru viata lor profesionala ulterioara ,iar elevii nostri care acum lucreaza
in diverse colturi ale globului in diverse profesii ne confirma ca nu ne
inselam.
Pentru ca RMM 8, va apare in apropierea sfarsitului de an dorim
tuturor colaboritorilor nostri “La multi ani”.

Presedintele Filialei Mehedinti a SSMR,


Profesor doctor Gh.Cainiceanu

-1-
EDITORIAL
Membrii Filialei Mehedinţi
a S.S.M.R.
- 31.XII.2007 –

1 Cainiceanu Gheorghe CNT PRESEDINTE


2 Prajea Manuela CNT Vicepresedinte
3 Stretcu Daniel Colegiul.Gh.Titeica Secretar
4 Grecu Vasile Colegiul Economic Casier
5 Nănuţi Dan CNT – ISJ Membru comitet
6 Nedeianu Dan Lic. Dl. Tudor Membru comitet
7 Săceanu Victor Şc. Gen. Nr.11 Membru comitet
8 Ungureanu Octavian Colegiul.Gh.Titeica Membru comitet
9 Gimoiu Iuliana CNT CENZOR
10 Giugiuc C-tin C. Dl. Tudor CENZOR

1 Istodor Oana Lic.S. Cioculescu 41 Pit Rada Marica Gen.5


2 Antonie Rodica CNT 42 Lupu Adrian Decebal
3 Giugiuc Leonard CNT 43 Nitoiu Angela Decebal
4 Popescu Eleodor CNT 44 Lugoj Tanta Decebal
5 Paponiu Dana CNT 45 Oprita Manuela Decebal
6 Crisan Livia Halanga 46 Bejenaru Laviniu CNT
7 Ionescu Adela C.U.Drobeta 47 Croitoru Ion CE
8 Fritea Eugen C. Dl. Tudor 48 Bizdoaca Claudia CE
9 Chilea Ion C. Dl. Tudor 49 Grecu Adela CE
10 Vasilcanu Florentina C. Dl. Tudor 50 Ladaru Damiela CE
11 Cristel Ecaterina C. Dl. Tudor 51 Stoican Victor Baia
12 Ciuca Ionel C. Dl. Tudor 52 Paulescu Petre Baia
13 Badescu Emilia C. Dl. Tudor 53 Barbulescu Marin Baia
14 Moclea Adriana Gen.11 54 Simionescu Loredana Baia
15 Lapadat Petruta Gen.7 55 Falon Florica Marina
16 Capraru Dorel Gen.7 56 Petrache Elena Marina
17 Pop Veronica Severinesti 57 Bogdan Daniel Marina
18 Pandioniu Aristita Hinova 58 Deris Antoaneta Lic.4
19 Diaconescu Emilia Gura Vaii 59 Varzaru Mariana Lic.Auto
20 Bondoc Gabriela Lic.Auto 60 Vaduva Ion Liv.Paulian
21 Pasov Nicoleta Lic.Auto 61 Untaru Ilie Odoblrja
22 Bondoc Lucian Tribunal 62 Popescu Marcel Odobleja
23 Grecu Luminita C.U.Drobeta 63 Ticusi Ovidiu Odobleja
24 Calafeteanu Gh Gen.9 64 Presneanu Doru Odobleja
25 Florescu Violeta Gen.4 65 Baloi Valeria Odobleja
26 Cărbunaru Dumitru Gen.4 66 Balu Nicoleta Odobleja
27 Ianasi Ion Gen.4 67 Balasoiu Daniela Lic.S. Cioculescu
28 Mălineanu Gabriela Grn.4 68 Raducan Emilia Decebal
29 Sitaru Dan CE 69 Pupaza Ecaterina Decebal
30 Blidaru C-Tin Malovat 70 Farago Alexandru T.Lalescu
31 Fluerasu Anghel Izv.Barzii 71 Bobic Nicolae T.Lalescu
32 Budanescu Lidia Sc.1 72 Gorun Sanda T.Lalescu
33 Stioiu Petre Bistrita 73 Drula Ileana Strehaia
34 Marin Felicia Sc.9 74 Juganaru Ion Strehaia
35 Zaharia Marius Ilovat 75 Neamtu Elian CE
36 Saftoiu Gheorghe Sc.15 76 Dragotescu Alexa Sc.7
37 Vasile Tomita C.U. 77 Tomoescu Loredana Sc.7
38 Bala Dumitru C.U. 78 Patrascoiu Constantin C.U.
39 Stuparu Dragos C.U. 79 Ionica Constantin Gen.14
40 Pit-Rada Ionel Vasile CNT

-2-
Petre Sergescu SSM
Concursul Interjudeţean de Matematică H
“PETRE SERGESCU”
Ediţia a IV-a, martie 2008, Drobeta Turnu-Severin
Prof.dr.Prajea Manuela ,Director Colegiul National Traian
Prof.dr.Cainiceanu Gheorghe,Presedintele Filialei Mehedinti a SSMR

Concursul Interjudetean de matematica “Petre Sergescu” editia a IV-a, s-a desfasurat la


Drobeta Turnu-Severin pe 21.03.2008. Organizatorii concursului, ca si la celelalte trei editii au fost
Colegiul National Traian din Drobeta Turnu-Severin si Filiala Mehedinti a SSMR.
La concurs au participat peste 500 de elevi din Mehedinti ,Caras-Severin, Gorj, Dolj, iar la
Simpozionul desfasurat cu aceasta ocazie au participat cu lucrari de istoria matematicii si articole
metodico-stiintifice peste 60 de cadre didactice din Mehedinti, Gorj, Bucuresti, Timis, Dolj, Caras-
Severin. Ne- a bucurat participarea din partea Biroului Consiliului National al SSMR a domnului
Mircea Trifu, care in cadrul Simpozionului a sustinut o interesanta prezentare despre Matematica in
limba romana din Ardeal.

SUBIECTE
Clasa a IV-a
{ } { }
1.a) Se consideră numerele : a = 1 + 2 ⋅ 1 + 2 ⋅ ⎡⎣1 + 2 ⋅ (1 + 2 ) ⎤⎦ şi b = 8 + 7 + 6 ⋅ ⎡⎣ 5 + 4 ⋅ ( 3 + 2 − 1 ) ⎤⎦ :9 .
Să se calculeze ( a + b ) :( a − b ) .
b) Suma a patru numere naturale impare consecutive este egală cu 2008. Să se afle numerele.
Ştefan Marica
2. Suma a trei numere naturale este egală cu 179. Al doilea număr este cu 5 mai mare decât dublul
primului număr şi cu 4 mai mic decât cel de-al treilea număr. Să se afle cele trei numere.
prelucrare G.M. 9/2007
3. Se consideră tabloul cu 100 de linii:
1
2 1
3 2 1
4 3 2 1
5 4 3 2 1
6 5 4 3 2 1
………………………..
100 99 98 97 ……..3 2 1
a) să se afle suma numerelor de pe linia a 10-a ;
b) de câte ori apare în tablou numărul 13 ?
c) să se determine numărul de apariţii al cifrei 7 în scrierea tuturor numerelor din tabloul considerat.
Manuela Prajea
Clasa a V -a
1.a) Aflaţi restul împărţirii numărului A= 1∙2∙3∙4∙ ........ ∙ 2007+2008 la 2002
b) Fie A={x | x=n +2007n+2009,n ∈ N} şi B={x|x=2007 2008n-1, n ∈ N}.Să se determine A ∩ B.
2

Raducan Emilia
2.Să se determine numerele naturale nenule m şi n astfel încât numărul m!+22n+1 să fie pătrat
perfect,unde m!=1∙2∙3∙.....∙m.
GM Nr.11/2007
3)Un elev are la inceputul anului şcolar 7 pixuri şi 20 de creioane.Fiind foarte neglijent el pierde in
fiecare zi câte două instrumente de scris din cele de mai sus.Dacă pierde două instrumente de
acelaşi fel ,părinţii îi mai cumpără , în aceeaşi zi, un creion ,iar dacă pierde un pix şi un creion îi
mai cumpără un pix.Care este ultimul instrument de scris ce îi rămâne elevului?
Lupu Adrian

-3-
Petre Sergescu
Clasa a VI-a
1) Să se determine numerele naturale de forma abc care satisfac relaţia:
a 3 (a 3 + b 3 + c 3 ) = 2008 .
Paponiu Dana
3n + 4
2) a)Să se arate că fracţia este ireductibilă pentru orice n ∈ N .
2n + 3
b)Să se determine numerele naturale m si n pentru care are loc relaţia:
3n + 4 m + 1
+ ∈N .
2n + 3 2m + 1
Marica Stefan
3) Fie segmentul [ AB] , A ≠ B şi 2007 puncte distincte, situate în interiorul său,
care îl împart în 2008 segmente. Punctului A i se asociază numărul 0 , iar
punctului B i se asociază numărul 1 . Celor 2007 puncte li se asociază arbitrar
unul din numerele 0 sau 1 .
Să se arate că, dintre cele 2008 segmente, un număr impar au extremităţile
asociate cu acelaşi număr.
Prajea Manuela
Clasa a VII-a
1.a)Demonstrati ca ( n + 6)(n + 7 )(n + 8)(n + 9) + 1 ∈ Q, oricare ar fi n numar natural;
b)Exista numere naturale n pentru care 3n + 2 ∈ Q ?
Rodica Antonie
2.a)Fie ABCD un trapez cu bazele AB si CD, AB=b, CD=a, 0<a<b, AC I BD={O}.
Dreapta d trece prin punctul O si intersecteaza bazele [CD] si [ AB] in punctele E,
respectiv F.Notam CE=x.Sa se determine valoarea lui x pentru care
aria(ADEF)=aria(BCEF) ;
b)Paralelogramul MNPQ este decupat din interiorul paralelogramului ABCD.Sa
se determine o dreapta care imparte figura ramasa in doua figuri cu aceeasi arie.
Justificati raspunsul.
Gheorghe Cainiceanu
3.Pentru n natural, se considera numarul real:
an= n + 1 + n ⋅( n +1 +1 − n 1 −1 )
a) calculati a2;
b) calculati partea intreaga a numarului an , n ∈ N;
G.M. nr.4/2007
Clasa a VIII-a
1. a)Se consideră mulţimea: A = { a+ }
b a, b ∈ N, 1 ≤ a < b ≤ 100 . Câte elemente are A I Q ?
b)Fie [ AB] şi [CD ] două segmente necoplanare, M mijlocul lui [ AB] şi N mijlocul lui [CD] . Să se
arate că dacă AC ⊥ CD şi BD ⊥ CD , atunci MN ⊥ CD .
Gimoiu Iuliana
2. Se consideră numerele reale nenule x şi y , pentru care numerele
x
, xy şi y x 2 + ( x + 1) + x 2 (x + 1)
2 2

y
sunt toate numere raţionale. Arătaţi că x şi y sunt de asemenea numere raţionale.
Nedeianu Dan
3. Se consideră un cub cu muchia 1cm şi un plan care secţionează cubul după un hexagon.
Să se determine valoarea minimă a perimetrului hexagonului.
Prajea Manuela

-4-
Petre Sergescu SSM
Clasa a IX-a H
1. Se considera cercul C(O;R) si punctele distincte A,B,C,D ∈ C(O;R).Daca
OA + OB + OC + OD = 0 sa se arate ca A,B,C,D sunt varfurile unui dreptunghi.
Manuela Prajea
2. a). Intr-o zi un pescar a pescuit 200 de platici si 18 salai. A doua zi a prins cu 3 platici mai
putin si cu 2 salai mai mult si tot asa, in zilele care au urmat cate 3 platici mai putin si cate 2
salai mai mult pana in ziua in care a prins mai multi salai decat platici. Cate platici a prins in
total?
b) Sa se demonstreze ca intre lungimile laturilor unui triunghi oarecare exista inegalitatea:
a b b c c a
− + − + − < 1.
b a c b a c
Este adevarata inegalitatea daca a,b,c sunt numere strict pozitive oarecare?
GheorgheCainiceanu ,Eleodor Popescu
3. Se considera functia f: R → R;
f ( x) = min(| x |,3, | 2 x − 1 |)
a). Sa se reprezinte grafic functia f.
b). Sa se rezolve ecuatia f(x)=2.
c). Sa se discute, in functie de parametrul real m, numarul solutiilor reale al ecuatiei f(x)=m.
d). Daca A = [1,2] ∪ {3,4 } si B = {1,2} ∪ [3,4] sa se reprezinte intr-un reper cartezian
produsele carteziene A X B si B X A.
.Daniel Sitaru ,Dan Nanuti
Clasa a X-a
1. Să se rezolve ecuaţia:
3
7 x − 6 + 3 x 2 − 10 x + 8 = 3 x 2 − 3 x − 6 + 2 .
G.M. nr. 11/2007
2. a) Dacă 0 < x < y şi a > 1 , să se arate că:
x ⋅ ax− y < y
b) Să se rezolve ecuaţia: 2cos 2 x = tgx .
Dan Nanuti
3. Fie, a, b ∈ C , a ∈ C − R , Im a ≠ Im b . Dacă numerele complexe a, b − a, b + a au acelaşi
modul, să se arate că b=0.
Nedeianu Dan
Clasa a XI-a
1). a). Calculati unde , a ∈ R si n ∈ .

b). Fie A ∈ (C), A≠ a. i. cu p≥5, p prim. Calculati .


Eleodor Popescu ,Constantin Giugiuc
2). Consideram n ∈ N , n≥2.
a). Aratati ca ecuatia are solutie unica in intervalul (0,1).
b).Pentru n≥2, notam cu solutia din intervalul (0,1) a ecuatiei .
Calculati lim x n .
n→∞

Cristinel Mortici
3). Definim functiile astfel:
.
a).Demonstrati relatiile:

-5-
Petre Sergescu
sh ( x)
b). Aratati ca functia f este inversabila si , apoi calculati lim .
x →0 x
c). Fie sirul definit astfel:
Calculati lim 4 (a n − 2) .
n
n →∞

Leonard Giugiuc
Clasa a XII-a
ε
1
1. Să se calculeze limita: l = lim ∫ sin
2007
xdx
ε →0 ε 2008
0
Eleodor Popescu
2. Fie f: [0,1] → [ 0, ∞ ) o funcţie continuă.
1 n k ⎛k ⎞
Demonstraţi că dacă şirul an = ∑ f ⎜ ⎟ este convergent la zero, atunci f(x) = 0, oricare
n k =1 k + 1 ⎝ n ⎠
ar fi x ∈ [0, 1].
Cristinel Mortici
⎧⎪⎛ ^ ^
⎞ ^ ^ ⎫⎪ ⎛ ^ ^

3. Se consideră mulţimea M = ⎨⎜ a^ b^ ⎟ | a, b ∈ Z 7 ⎬ şi matricea A = ⎜ 4^ 3^ ⎟ ∈ M .
⎪⎩⎜⎝ b a ⎟⎠ ⎪⎭ ⎜ 3 4⎟
⎝ ⎠
Să se rezolve în M ecuaţia X6 = A.
Manuela Prajea
Premiul I
XII Gosea Victor CNCarol,Craiova, Stoian Bogdan CN G.Cosbuc,Motru, Tigora Andrei CN Traian,Drobeta XI Dragoi
Maris CN G.Cosbuc,Motru, X Pasureanu Victor CN Carol.Craiova, IX Tesila Bianca CN Traian,Drobeta, VIII Nistor
Adriana Gen.14.Drobeta, VII Stefan Andrei CN Traian,Drobeta, VI Puican Tiberiu CN Traian,Drobeta, Trofin Raluca
CN Carol Craiova, V Pogacean Victor CN Traian,Drobeta, IV Burtea Catalin Gen.2,Drobeta, Popescu Cristina
Gen.16,Drobeta
Premiul II
XII Mituca Atena CN Traian,Drobeta, XI Butaru Nicu CN Traian Drobeta, Posa Bogdan, CN G.Cosbuc,Motru,
X Pirvulescu Dan CN Traian,Drobeta, IX Mirosu Raluca CNTraian,Drobeta, VIII Baleanu Andrei CNG.Cosbuc,Motru,
VII Tanasie Denisa CNTraian,Drobeta, VI Troncota Diana CNTraian, V Calota Dragos CN Carol,Craiova, IV Paleacu
Cosmin Gen.2,Drobeta, Sontea Claudiu Gen.14
Premiul III
XII Coanda Oana CNTraian,Drobeta, Prundeanu Andreea CNTraian,Drobeta, XI Ciorobea Mihai ,Mema
Alexandra,Nistor Ovidiu CNTraian,Drobetam, X Birovescu Georgiana CNTraian Drobeta ,Alexandru Bogdan CN
Carol Craiova, IX Agape Mihai CNTraian,Drobeta, VIII Andreescu Madalina CNTraian,Drobeta, Semenescu Anca
CDLoga,Caransebes, VII Anghel Cristian V Gomoiu ,Vinju-Mare, Gimoiu Ruxdandra CNTraian Drobeta, VI Benga
Andrei CNTraian Drobeta, V Chirita Dan CNTraian,Drobeta, IV Butaru Madalina Gen.14,Drobeta,Craciun Madalina
Gen.2, Drobeta, Dragotescu Radu Gen.6,Drobeta, Filip Radu Gen.6,Drobeta, Lica Robert Gen.2,Drobeta, Tanasie
Danut Gen.2,Drobeta, Balu Smaranda Gen.2, Drobeta

-6-
Petre Sergescu SSM
SIMPOZIONUL NATIONAL PETRE SERGESCU 21.03.2008 H
EDITIA A IV-A
MIRCEA TRIFU Secretar general al SSMR,Bucuresti
Aritmetica in limba romana din Ardeal
CHIRCU MARIANA DOINITA Inspector general ISJ
Matematici elementare : aspecte metodice prin aplicatii practice
CAINICEANU GHEORGHE, CNT,
Asupra numerelor algebrice
PRAJEA MANUELA CNT,
Un criteriu de tip L’Hospital pentru calculul unor limite de functii
STOICA RODICA CNT,
Stiinta si constiinta-moralitatea gandirii creatoare
GHIMES DANIELA Inspector ISJ Mehedinti
Din viata si activitatea matematicianului roman Traian Lalescu
ANTONIE RODICA MIHAELA CNT,
Triunghiul ortic
PAPONIU DANA ,CNT,
Polinoame ireductibile
MARICA STEFAN Prof.Pensionar CNT, Colegiul National Traian
In amintirea unor fosti elevi
DAN NEDEIANU Colegiul Tehnic D-l Tudor
Aplicatii ale teoremei Lagrange in rezolvarea unor ecuatii exponentiale
GIMOIU IULIANA CNT
Proprietatea lui Darboux
VIOREL SAHAGIA Profesor pensionar CNT
Aspecte privind generalul si particularul in matematica
LUPU ADRIAN Col.Tehnic Decebal
Congruente modulo n in gimnaziu si liceu
GHIOCEL FLORIN Inspector ISJ
Matematicieni europeni din mileniul III
GHIOCEL GEORGETA GSI-AUTO
Mobilitate europeana prin programe comunitare
GIUGIUC LEONARD CNT
Aplicatii ale criteriului Cesaro-Stoltz
GIUGIUC CONSTANTIN Col.Tehnic D-l Tudor
Metode de rezolvare pentru determinanti
ANTONIE NICUSOR Lic.Stefan Odobleja
Aplicatii ale calculului integral in fizica
IONICA CONSTANTIN Sc.Gen.14
Metoda reducerii la absurd
BEJENARU LAVINIU CNT
Siruri cu aspecte speciale
BALOI VICTORIA,BALU NICOLETA,Lic.Pedagogic St.Odobleja,
Vocatia formatoare a scolii
MIHAELA POPESCU Institutor Sc.Gen.2,
POPESCU DUMITRU Inst.Sc.Gen.24 Craiova
Noi modalitati de abordare a lectiei de matematica
PIT-RADA MARICA Sc.gen.Nr.5
O metoda de calculare a distantei dintre doua drepte neparalele in spatiu
PIT-RADA VASILE CNT
Despre spatii proiective si teoria codurilor
PIPINA CAMENITA Institutor Sc.Gen.3
Gheorghe Titeica-matematician de seama al judetului Mehedinti
ALINA TEIS Inst.Liceul Pedagogic Stefan Odobleja
Jocul didactic, modalitate de optimizare a procesului de predareinvatare la ciclul primar
GORUN SANDA Lic.Traian Lalescu, Orsova
Probleme cu continut practic de aflare a extremelor
OPRITA MANUELA Col.Tehn.Decebal
Teoreme din geometrie demonstrate trigonometric
MIHAIL MARIA,MIHAIL ANDREI, Institutori Sc.Gen.6
Stimularea creativitatii prin orele de matematica

-7-
Petre Sergescu
COCOARA VICA Obarsia Closani
Aspecte metodice in rezolvarea unor probleme de aritmetica
OSIAC VIOREL Inginer SC Termo. SA
Dan Barbilian- intre matematica si poezie
PLOTOGEA JANETA ,Col.Tehnic Decebal
Polinoame ireductibile cu coeficienti intr-un corp
BONDOC GABRIELA Grup Scolar Auto
Cateva considerente despre functii monotone si continue
LUGOJ EUGEN CNT
Interdisciplinaritatea matematica-chimie in rezolvareaproblemelor de chimie
NANUTI DAN CNT, SITARU DAN Col.Economic
Caracterizarea structurilor de Grupuri finite utilizand numarul de automorfisme de corp
ROATES MISU CNT,ROATES OLGA CNT
Matematica si adolescenta
NISTOR CORNELIA CNT
Matematica aplicata in biologie
RADUCAN EMILIA STEFANIA Col.Tehn.Decebal
Structuri algebrice. Grupuri.
BIZDOACA CLAUDIA Col.Economic
Utilizarea polinoamelor ciclotomice in rezolvarea ecuatiilor algebrice cu coeficienti complecsi
HRUBY ELISABETA CNT
Statistica matematica si literatura
TOMA NINETA CNT,
Poezia –geometrie a formelor
OSAIN VICTORIA Strehaia
Patrate si cuburi de numere intregi
TRUSCA DANIELA CNT
Paradoxuri si probabilitati in poezie
BADESCU OCTAV CNT
Mituri si cauze in schimbarile climatice
LUGOJ TANTA Col.Tehnic Decebal
Aspecte metodice privind Descoperirea, motivarea si interpretarea inegalitatii lui Cauchy si
a coeficientului de corelatie cu aplicatii in diverse domenii
ZAGARA GABRIELA CNT
Limbaje formale
ZAICU RODICA, DRAGHIA MADALINA Institutori CN G.Cosbuc Motru
Modalitati de optimizare a tehnologiei didactice la matematica
LICA MARIELA CAMELIA CNT
Matematica-Muzica a ratiunii
CONSTANTINESCU CERASELA CNT
Mari matematicieni romani
CIVITU LUISA CNT
Petre Sergescu-personalitate multilaterala
FLEANCU RALUCA CNT
Petre Sergescu-ambasador de renume al Romaniei in lume
TRAILESCU GOGAN DIANA VERONICA ,Obarsia Closani
Teorema lui Sylvester si aplicatii
COVASALA SORIN Obarsia Closani
Cercul lui Euler
POPESCU OCTAVIAN TUDOR Colegiul F.Nitti.Timisoara
Previziune economica prin metoda autoregresiva
SAMFIRESCU MIRCEA CNT
Sisteme dinamice deterministe.Haos determinist.
OPROIU MIHAELA CNT
Pitagora si pitagorismul
CATANA ELENA CNT
Aristotel-omul si opera
HUMITA DORINA,MIRULESCU MARITA, CD Loga,Caransebes
Grigore Moisil-O legenda a matematicii romanesti
MANDRESI ANA CD Loga Caransebes
Formarea si dezvoltarea competentelor prin studiul teoremelor de medie
RAESCU GHEORGHE,Orsova
Probleme deosebite de geometrie

-8-
Petre Sergescu SSM
GHINEA MARIA CNT H
Numere simbolice in crestinism
FARAGO AFRODITA Orsova
Matematicieni romani-Dimitrie Pompeiu
FARAGO ALEXANDRU Orsova
Traian Lalescu
DRULA ILEANA Strehaia
Inegalitati algebrice
DRULA MIHAI Strehaia
Bazele psihopedagogice si metodologice ale rezolvarii problemelor
MALINEANU GABRIELA Sc.Gen.4
Legarea teoriei de practica
CHIRFOT CARMEN VICTORITA Col.Tehnic D-l Tudor
Aplicatii care folosesc relatia lui Legendre pentru calcularea exponentului numarului prim
CRISAN LIVIA Col.Tehn.Lorin Salagean
Aspecte metodice privind predarea-invatarea proiectiilor ortogonale pe un plan
NITOIU ANGELA Col.Tehnic Decebal
Functia homografica
ROMAN CAMELIA CNT
Inteligenta artificiala
STIOIU FLOAREA,GRIGORAS ANISOARA Sc.Gen.14
Matematicieni celebri
MOCLEA ADRIANA Sc.Gen.11
Importanta constructiilor geometrice in rezolvarea problemelor de geometrie
ROMAN ANA Institutor Sc.Gen.11
Importanta rezolvarii problemelor de geometrie in ciclul primar
BASULESCU MARIA Institutor Sc.Gen.11
Formarea deprinderilor de rezolvare a problemelor la ciclul primar
ICA FLORENTINA Institutor I.St.Paulian
Introducerea notiunilor matematice si dezvoltarea abilitatilor logice prin utilizarea jocului
didactic la ciclul primar
MEDRAGONIU CRISTIANA Institutor,Gen.11
Rolul jocului didactic matematic in lectiile de matematica
BADOIU EMILIA CNT
Matematici financiare in invatamant
PAUNESCU LIVIA CNT
Elemente de statistica in organizarea bazelor de date
DRAGOMIR VETURIA CNT
Programe soft pentru calcul financiar
CIOBOTA NORICA CNT
Experimente practice de fizica matematica

-9-
NOTE MATEMATICE

Pregatire pentru olimpiadele, concursurile


şi barajele de matematică
prof.dr.Manuela Prajea,
Colegiul Naţional „Traian”
Prezenta rubrică vine în sprijinul elevilor care se pregătesc pentru olimpiade, concursuri
şi baraje la matematică, prin expunerea unor probleme inedite prin originalitatea
raţionamentului, date la concursurile naţionale sau regionale pe mapamond. Problemele
expuse sunt însoţite de soluţii comentate, menite să permită elevului să surprindă atât
mecanismul unui raţionament adecvat cât mai ales posibilitatea sugerării treptate a paşilor
logici, elevul fiind astfel invitat să găsească prin forţe proprii continuarea ideilor de rezolvare.
În acest sens am marcat corespunzător ideile care invită la „spargerea problemei”.

Se consideră mulţimea 2n = {1, 2,3,..., 2n − 1, 2n} şi submulţimile sale disjuncte


{a1, a2 , a3, ..., an } , {b1, b2 , b3, ..., bn } astfel încât a1 > a2 > a3 > ... > an şi b1 < b2 < b3 < ... < bn .
Să se arate că are loc egalitatea a1 − b1 + a2 − b2 + ... + an − bn = n 2 .(identitatea lui
Prozvolov)

Soluţie:
Partiţionarea mulţimii 2n conform enunţului sugerează raportarea termenilor secvenţelor
( ai )i =1,n şi ( bi )i =1,n la partiţia canonică X = {1,2,..., n} şi Y = {n + 1, n + 2,...,2n} .

Dificultatea constă în a observa că dacă ak ∈ X atunci bk ∈ Y şi dacă ak ∈ Y atunci bk ∈ X ,


k = 1, n .

Într-adevăr, dacă prin absurd, există ak , bk ∈ X atunci ak ≤ n, bk ≤ n .


În această situaţie, un număr de cel puţin n − k + 1 termeni ai secvenţei ( ai )i =1,n vor aparţine
mulţimii X şi analog un număr de k termeni ai secvenţei ( bi )i =1,n vor aparţine din nou
aceleaşi mulţimi X . Astfel mulţimea X va conţine cel puţin n + 1 numere naturale distincte -
contradicţie.
Analog, cazul ak , bk ∈ Y conduce la contradicţie.

Suntem în măsură să calculăm suma cerută.

Având în vedere cele de mai sus conchidem că pentru orice pereche ( ak , bk ) , unul dintre
numere aparţine mulţimii X şi celălalt mulţimii Y . Deci:
a1 − b1 + a2 − b2 + ... + an − bn = ⎡⎣( n + 1) + ( n + 2 ) + ... + 2n ⎤⎦ − [1 + 2 + ... + n ] = n + n + ... + n = n 2 .

În spiritul identitaţii Prozvolov propunem spre rezolvare următoarea :


Se consideră {a1 , a2 , a3, ..., an } , {b1 , b2 , b3, ..., bn } două submulţimi disjuncte ale mulţimii
{1, 2,...,2n} , n ∈ *
astfel ca a1 > a2 > a3 > ... > an şi b1 < b2 < b3 < ... < bn .
⎧ ak bk ⎫
Pentru orice k = 1, n definim α k = max ⎨ , ⎬.
⎩ bk ak ⎭
Să se arate că α1α 2 ...α n ∈ .

- 10 -
NOTE MATEMATICE SSM
H
Probleme pentru Olimpiada
elev. Ioana Ionas
prof. dr. Daniel Stretcu

Probleme propuse pentru clasa a VI-a

1. Arătaţi că oricare trei numere prime, mai mari sau egale cu 13, se pot alege
două astfel încât suma sau diferenţa lor să se dividă cu 12.

Soluţie:
Din teorema împărţirii cu rest, un număr prim, mai mare sau egal cu 13 este de forma
12k+1, 12k+5, 12k+7 sau 12k+11, cu alte cuvinte de forma 12k ± 1 sau 12k ± 5. Folosind
principiul cutiei, fiind date 3 numere prime, vor exista 2 numere din aceeaşi clasă a căror
diferenţă sau sumă este evident divizibilă cu 12.

2. Câte cifre are numărul cifrelor numărului 20082008 ?

Soluţie:
Notăm cu N(a) numărul cifrelor numărului a.
Avem: 1000<2008<10000 ⇒ 10 3⋅ 2008 < 2008 2008 < 10 4 ⋅ 2008 ⇒
⇒ N (103⋅ 2008 ) ≤ N (20082008 ) ≤ N (10 4⋅ 2008 ) ⇒
⇒ 6024 ≤ N(20082008 ) ≤ 8032 ⇒ N(6024) ≤ N( N(20082008 )) ≤ N (8032) ⇒
⇒ 4 ≤ N ( N( 20082008 )) ≤ 4
Deci N( N(2008 2008 )) = 4 .

3. Pe o tablă 3× 3 avem un singur pătrăţel negru într-un


colţ, iar celelalte sunt albe. Vom numi recolorare operaţia de
schimbare a culorilor în toate pătratele unei linii sau coloane.
Demonstraţi că tabla nu poate fi transformată într-o tablă
complet albă recolorând, una câte una, câteva linii şi câteva
coloane.

Soluţie:
Demonstrăm prin reducere la absurd. Presupunem că tabla ar putea
deveni toată albă după un anumit număr de recolorări. Punem în
pătrăţelele negre -1, iar în
pătrăţelele albe numărul 1. Luăm ca invariant produsul numerelor din
pătrăţelele haşurate în figura alăturată.

Iniţial, produsul este -1 şi după fiecare recolorare rămâne neschimbat. Deci, oricâte
recolorări am face nu poate fi egal cu 1, ca în cazul când toate pătrăţelele ar fi albe şi am
obţinut o contradicţie.

- 11 -
NOTE MATEMATICE

Probleme pentru Olimpiada


prof. dr. Daniel Stretcu

Probleme propuse pentru clasa a XI-a

1. Fie A∈M2 (R) o matrice pătratică ce are proprietăţile det( A − I 2 ) = 1 şi


det( A + I 2 ) = 3 .
Să se calculeze:
a) det( A − 4 ⋅ I 2 ) şi det( A + 4 ⋅ I 2 )
b) det( A 4 − 5 ⋅ A 2 + 6 ⋅ A)
Soluţie:

a) Fie P ( X ) = det( X ⋅ I 2 − A) = X 2 − X ⋅ a + b , unde P(X ) este polinomul caracteristic


al matricei A cu a = det A şi b = tr ( A) .
Folosind det( X ⋅ I 2 − A) = ( −1) 2 ⋅ det( A − X ⋅ I 2 ) = det( A − X ⋅ I 2 ) , avem
P(1) = det( A − I 2 ) = 1 − a + b . Dar det( A − I 2 ) = 1 , din ipoteză, rezultă a = b (1) . Analog
P(−1) = det( A + I 2 ) = 1 + a + b , iar det( A + I 2 ) = 3 . Deci a + b = 2 (2) .
Din (1) şi ( 2) ⇒ a = b = 1 ⇒ P ( X ) = X 2 − X + 1 .
det( A − 4 ⋅ I 2 ) = P(4) = 13 .
det( A + 4 ⋅ I 2 ) = P(−4) = 21.
b) Avem A 2 − A + I 2 = O2 (din Teorema lui Hamilton-Cayley).
A 2 = A − I 2 ⇒ A 4 = ( A − I 2 ) 2 ⇒ A 4 = A 2 − 2 ⋅ A + I 2 (3)
Folosind în det( A 4 − 5 ⋅ A 2 + 6 ⋅ A) relaţia (3) , rezultă
det( A 4 − 5 ⋅ A 2 + 6 ⋅ A) = det[( A 2 − 2 ⋅ A + I 2 ) − 5 ⋅ A 2 + 6 ⋅ A] = det(−4 ⋅ A 2 + 4 ⋅ A + I 2 ) =
= det[−4( A − I 2 ) + 4 ⋅ A + I 2 ] = det(5 ⋅ I 2 ) = 25 .

6xn + 1
2. Fie şirul ( x n ) n≥ 0 , definit prin relaţia x n+1 = , ( )n ∈ ℕ şi x0 > 1 .
5xn + 2
Să se arate că şirul ( x n ) n≥ 0 este convergent şi să se calculeze lim x n .
n →∞

Soluţie: Metoda I:

⎛6 1⎞
Fie A = ⎜⎜ ⎟⎟ matricea asociată şirului omografic ( x n ) n≥ 0 .
⎝5 2⎠
Din teorie cunoaştem că dacă λ1, λ2 sunt rădăcinile ecuaţiei:
λn − λn2 λn−1 − λn2−1
λ2 − tr ( A) ⋅ λ + det A = 0 (1) cu λ1 ≠ λ 2 atunci A n = 1 ⋅ A − det A ⋅ 1 ⋅ I2 .
λ1 − λ2 λ1 − λ2
⎛6 1⎞
A = ⎜⎜ ⎟⎟ ⇒ tr ( A) = 8 şi det A = 7 .
⎝5 2⎠
Relaţia (1) devine λ2 − 8λ + 7 = 0 ⇒ λ1 = 1 şi λ 2 = 7 .

- 12 -
NOTE MATEMATICE SSM
H
1− 7n 1 − 7 n−1 7n −1 7 n −1 − 1
An = ⋅ A−7⋅ ⋅ I 2 ⇒ An = ⋅ A− 7⋅ ⋅ I2
1− 7 1− 7 6 6
1 ⎛ 5 ⋅ 7n + 1 7n −1⎞ (5 ⋅ 7 n + 1) x 0 + 7 n − 1
A n = ⎜⎜ n ⎟ ⇒ x =
6 ⎝ 5(7 − 1) 7 n + 5 ⎟⎠
n
5 ⋅ (7 n − 1) x 0 + 7 n + 5
1 1
7 n [(5 + n ) x0 + 1 − n ]
(5 ⋅ 7 n + 1) x0 + 7 n − 1 7 7 = 5x 0 + 1 = 1 .
lim x n = lim = lim
n→∞ n → ∞ 5 ⋅ (7 − 1) x + 7 + 5
n n n→∞ n 1 5 5x 0 + 1
0 7 [5(1 − n ) x0 + 1 + n ]
7 7

Metoda a II -a

Determinăm an, b n, cn, dn astfel încât:


a n ⋅ x 0 + bn ⎧a = 1, b0 = 0, c0 = 0, d 0 = 1
xn = cu ⎨ 0
cn ⋅ x0 + d n ⎩ a1 = 6, b1 = 1, c1 = 5, d 1 = 2
a n ⋅ x 0 + bn
6⋅ +1
6xn + 1 c n ⋅ x0 + d n 6 a n ⋅ x0 + 6b n +c n ⋅ x0 + d n
x n+1 = ⇒ x n+1 = ⇒ x n+1 = ⇒
5xn + 2 a ⋅ x + bn 5a n ⋅ x0 + 5b n +2cn ⋅ x0 + 2 d n
5⋅ n 0 +2
cn ⋅ x0 + d n
x0 (6 a n + c n ) + 6bn + d n ⎫ ⎧ a n+1 = 6a n + cn
x n+1 =
x0 (5a n + 2c n ) + 5bn + 2d n ⎪⎪ ⎪⎪ bn+1 = 6bn + d n
⇒ ⎬⇒⎨
x n+1 =
a n +1 ⋅ x0 + bn+1 ⎪ ⎪ c n+1 = 5a n + 2c n
c n+1 ⋅ x 0 + d n+1 ⎪⎭ ⎪⎩d n +1 = 5bn + 2d n
⎧ 1
⎪ an = (5 ⋅ 7 n + 1)
6
⎪ 1
⎪ bn = (7 n − 1)
⎪ 6
Rezolvând acest sistem ⇒ ⎨
5
⎪ cn = (7 n − 1)
⎪ 6
⎪ 1 n
⎪⎩ d n = (7 + 5)
6
(5 ⋅ 7 + 1) x0 + 7 − 1
n n
xn =
5 ⋅ (7 n − 1) x0 + 7 n + 5
1 1
7 n [(5 + ) x0 + 1 − n ]
(5 ⋅ 7 n + 1) x0 + 7 n − 1 7 n
7 = 5x 0 + 1 = 1
lim x n = lim = lim
n→∞ n → ∞ 5 ⋅ (7 n − 1) x + 7 n + 5 n→∞ 1 5 5x 0 + 1
0 7 n [5(1 − n ) x0 + 1 + n ]
7 7

- 13 -
NOTE MATEMATICE

ASUPRA NUMERELOR ALGEBRICE


Prof.dr. Gheorghe Cainiceanu,CNT

INTRODUCERE
Analizand numerele reale sau complexe dupa criteriul de a fi sau a nu fi radacinile unor
polinoame, C.H.Hermite (1822-1901) si F.Lindemann (1852-1939) au introdus notiunile de
numere algebrice si numere transcendente. Nota de fata isi propune prezentarea acestor
numere si a unor exemple accesibile la nivel elementar.

NOTIUNILE DE NUMAR ALGEBRIC SI NUMAR TRANSCENDENT


DEFINITIA 1 Daca un numar a ∈C este radacina a unei ecuatii de tipul
xn+a1xn-1+…+an =0
cu coeficienti rationali, a se numeste numar algebric.
EXEMPLE 1) orice numar rational a este algebric deoarece este solutia ecuatiei
x-a=0.
2) 2 este algebric deoarece este solutie a ecuatiei x2-2=0 ,ecuatie cu coeficienti rationali.
In 1873 C.H.Hermite demonstreaza primul ca e (baza logaritmului natural) nu este numar
algebric.
Ne propunem aici sa aratam doar ca e nu este numar rational.
PROPOZITIE numaeul lui Euler ,e, nu este rational.
Demonstratie. Pentru sirul
1 1 1
s n = 1 + + + ..... +
1! 2! n!
Se arata prin calcule ca 0<e-sn =
1 1 1 1 1 1
+ + .... < [1 + + + ....] = , n ≥ 1.
(n + 1)! (n + 2)! (n + 1)! n + 1 (n + 1) 2
n!n
Deci
1 1 1 φ
e = 1 + + + ..... + + n , φ n ∈ (0,1) .
1! 2! n! n! n
p
Presupunem acum ca e = .Deducem ca exista φ (q ) ∈ (0,1) ,pentru care avem
q
p 1 1 1 φ (q)
= 1 + + + ... + .
q 1! 2! q! q!q
Prin inmultirea convenabila, gasim
1 1
φ (q) = q! p − (1 + + ... )q!q ∈ Z , ceea ce este absurd.
1! q!
In 1882 F.Lindemann a demonstrate ca π nu este algebric.
DEFINITIE. Daca numarul complex a nu este algebric el se numeste transcendent.
OBSERVATIE O consecinta a faptului ca π este transcendent, este aceea ca nu putem
construe cu rigla si compasul un segment egal cu lungimea cercului, sau un patrat echivalent
cu un cerc.
PROPOZITIE Numerele algebrice formeaza un corp in raport cu operatiile de adunare si
inmultire induse di multimea numerelor complexe.
DEFINITIE Fie K⊂ L corpuri commutative astfel incat K este subcorp al lui L. spunem ca L
este o extindere a lui K.
EXEMPLU C este o extindere a lui R.

- 14 -
NOTE MATEMATICE SSM
H
PROPOZITIE Fie K un corp comutativ si f ∈K[X] ,grad f≥1. Exista o extindere a lui K ,
notata L, astfel incat f sa aiba cel putin o radacina in L.
Demonstratie [1].
COROLAR Fie K un corp comutativ si f ∈K[X] ,grad f ≥ 1.Atunci exista o extindere a lui K
in care f sa aiba toate radacinile.
DEFINITIE se numesc corpuri de numere algebrice extinderile finite ale corpului numerelor
rationale, iar elementele lor se numesc algebrice.
DEFINITIE Un numar algebric se zice intreg daca satisface o ecuatie algebrica monica cu
coeficienti intregi.

EXEMPLE
π
1) Numarul ctg este intreg algebric.
8
π
Justificare.Prin calcul elementar si cunoscut, deducem ca ctg = 2 + 1 .O ecuatie cu
8
coeficienti intregi care admite acest numar ca solutie este ecuatia
x2-2x-1=0.
1
2) Numarul este intreg algebric.
π
sin
8
Justificare. Se arata ca acest numar este solutie a ecuatiei x4-8x2+8=0, deci este numar intreg
algebric.

BIBLIOGRAFIE
[1] C.Nastasescu,C.Nita,C.Vraciu Bazele Algebrei Editura Academiei 1986
[2] J.Kurschak Probleme de matematica pentru concursurile din Ungaria, EDP, 1972
[3] Z.I.Borevici, I.R.Safarevici, Teoria Numerelor, Ed.St.Enciclopedica,Bucuresti 1985.
[4] Gh.Siretchi Calcul diferential si integral , Ed.St.Enciclopedica, Bucuresti 1985.

- 15 -
CERCUL DE MATEMATICA
Tema pentru grupa de performanta la clasa a- V -a

Operaţii cu numere naturale binare


Prof. Chirfot Carmen – Victoriţa
Prof. Chilea Ion
Colegiul Tehnic „Domnul Tudor”
În această notă, vom enumera şi exemplifica câteva dintre operaţiile de bază care se pot
aplica numerelor scrise în baza 2. Aşa cum ştim, cifrele bazei de numeraţie 2 sunt 0 şi 1.
Operaţiile de adunare, scădere, înmulţire şi împărţire sunt similare celor din baza de numeraţie 10,
cu specificaţia că se va ţine cont de câteva reguli enumerate în acest articol.
1. Transformarea unui număr natural din baza 10 în baza 2
Se împarte numărul natural n la 2, apoi câtul obţinut se împarte din nou la 2 şi algoritmul
continuă până când obţinem câtul 1. Se reţine ultimul cât şi resturile obţinute la toate aceste
împărţiri în ordine inversă, forma obţinută reprezentând scrierea în baza 2 a numărului n.
2. Transformarea unui număr natural din baza 2 în baza 10
Să presupunem că numărul natural scris în baza 2 este
n = a1 a 2 a 3 ...a p ( 2) , a i ∈ { 0, 1}, i = 1, p , a1 ≠ 0, p ∈ N * . Atunci, în baza 10 avem:

n = a1 ⋅ 2 p −1 + a 2 ⋅ 2 p − 2 + ... + a p −1 ⋅ 2 + a p , a i ∈ { 0, 1}, i = 1, p, a1 ≠ 0, p ∈ N * .

3. Reguli de adunare în baza 2


0 + 0 = 0; 0 + 1 = 1; 1 + 0 = 1; 1 + 1 = 10 2 .
4. Reguli de scădere în baza 2
0 − 0 = 0; 1 − 0 = 1; 1 − 1 = 0; 10 ( 2) − 1 = 1 .

5. Reguli de înmulţire în baza 2


0 ⋅ 0 = 0; 0 ⋅ 1 = 1; 1 ⋅ 0 = 1; 1 ⋅ 1 = 1 .
6. Reguli de împărţire în baza 2
0 : 1 = 1; 1 : 1 = 1 . Împărţirea la 0 nu poate fi efectuată,
73 2
aşa cum nici în baza 10 nu există (de fapt, nu există în nici
72 36 2
o bază de numeraţie).
=1 36 18 2
=0 18 9 2
Exemple şi exerciţii
=0 8 4 2
a) Transformaţi în baza 2 numărul 73.
1 4 2 2
Soluţie: Împărţim repetat numărul natural 73 la 2. Se reţine
0 2 1
ultimul cât şi resturile obţinute la împărţiri se citesc de la
dreapta la stânga. Prin urmare, 73(10) = 1001001( 2) . 0

b) Transformaţi numărul natural 1001001( 2) în baza 10.

-1616 -
CERCUL DE MATEMATICA SSM
Soluţie: 1001001( 2) = 1 ⋅ 2 + 0 ⋅ 2 + 0 ⋅ 2 + 1 ⋅ 2 + 0 ⋅ 2 + 0 ⋅ 2 + 1 = 64 + 8 + 1 = 73 .
6 5 4 3 2
H
c) Calculaţi 1111( 2 ) + 11( 2) .

Soluţie: Vom efectua adunarea în scris, asemenea celei din baza 10, dar ţinând cont de regulile
• • •
1 1 1 1( 2 ) +
de adunare în baza 2: Observaţie: Punctele scrise deasupra
1 1( 2 )
1 0 0 1 0 ( 2)
cifrelor binare sunt acele cifre pe care le ţinem în minte, atunci când rezultatul depăşeşte cifra
maximă în baza 2 şi aceste puncte sunt de valoare egală cu 1 şi se vor aduna cifrelor de pe
coloana asociată acestora.

d) Calculaţi 1001( 2 ) − 11( 2) .

→ →
1 0 0 1( 2) −
Soluţie: Observaţie: Săgeţile de deasupra cifrelor binare
1 1( 2)
= 1 1 0 ( 2)

desemnează ca s-a făcut un împrumut de o unitate la cifra care are săgeată deasupra, deci din
cifra de sub săgeată se scade 1 în baza 2.
e) Calculaţi 1111( 2) ⋅ 11( 2 ) . Exerciţii propuse pentru rezolvare
111001( 2) + 111( 2)
1) Efectuaţi următoarele operaţii ;
111001( 2) − 111( 2)
1 1 1 1( 2) ⋅
1 1( 2) 111001( 2) ⋅111( 2)
.
101010( 2 ) : 111(2 )
1 1 1 1( 2) +
Soluţie:
1 1 1 1( 2) 2) Rezolvaţi ecuaţia
1 0 1 1 0 1( 2) [ x ⋅ 101( 2) + 1001( 2 ) ] ⋅ 11( 2) + 100 ( 2) = 10001000( 2) , fără a

f) Calculaţi 1001( 2) : 11( 2) . face transformări în baza 10.


3) Transformaţi în baza 2 numărul 256.
Soluţie:
1001 (2) 11 (2) 4) Transformaţi în baza 2 numărul 2 n , n ∈ N .

11 11 (2) 5) Transformaţi în baza 10 numărul 1000


142 43, n ∈ N .
...000
de n ori 0
= =11
6) Generalizând, efectuaţi 34 ( 7 ) + 33 (7 ) , fără a face
11
== transformări în baza de numeraţie 10.

- 17 -
CERCUL DE MATEMATICA
Tema pentru grupa de performanta la clasa a- V -a

Prof. Adi Lupu, Dr. Tr. Severin

1. Să se afle n Є N astfel încât n 2 + 20n + 99 = 2008 2008 + 2009 2009


Soluţie:
M4
20082008 + 2009 2009 = M 4 + ( M 4 + 1) 2009 = M 4 + M 4 + 1 = M 4 + 1. Ecuaţia se rescrie:

n 2 + 20n + 100 = 20082008 + 20092009 + 1 ⇒ (n + 10) 2 = M 4 + 1 + 1 ⇒ (n + 10) 2 = M 4 + 2, care nu


are soluţie, pentru că un pătrat perfect nu poate fi decât de formele M4 sau M4+ 1.

2. Rezolvaţi in multimea numerelor naturale ecuaţia 2008 x = y 2008 ( y 2008 + 2007) + 2007
Soluţie:
Observăm că pentru x = 0 nu se obţin soluţii. Pentru x Є N* numărul 2008x este par. Cum
y2008 şi y2008+2007 au parităţi diferite => y2008(y2008+2007) + 2007 este impar.
În final: S = Ø

3. Rezolvaţi in Z ecuatia: 20082007 ...... 3210 – 4 = n 2008(n2008 + 4)


Soluţie:
Ecuaţia se rescrie: 20082007 ...... 3210 = (n2008 + 2)2
Membrul stâng se divide cu 10, dar nu cu 100, deci nu este pătrat perfect.
În final S = Ø.

4. Dacă ma, mb, mc sunt lungimile medianelor triunghiului oarecare ABC de laturi AB = c, AC
m a + mb + m c
= b, BC = a, să se arate că fractia este subunitară.
a+b+c
Soluţie:
Prelungim mediana AM a triunghiului ABC (M Є (BC)) cu segmentul MA`=AM. În
paralelogramul ABA`C rezultă că AB = A`C.
b+c b+c
În triunghiul AA`C: 2AM < AC + CA` => AM < => ma < .
2 2
a+c a+b m + mb + mc
Analog mb < , mc < => ma + mb + mc < a + b + c => a <1
2 2 a+b+c

-1818 -
CERCUL DE MATEMATICA SSM
H
5. Într-un triunghi oarecare lungimile laturilor sunt numerele a,b,1, unde a,b Є N*. Să se arate
că a – b se divide cu 20092008.
Soluţie:
Avem 0 ≤ |a - b| < 1. Deoarece a,b Є N* ⇒ |a-b| Є N. Deci |a - b| = 0 => a – b = 0 =>
(a – b) M 2009 2008

G.1. Să se arate că dacă a1, a2, …, a2009 sunt numere naturale astfel încât a1+a2+…+a2009 este par
atunci a12 + a 22 + ... + a 2009
2
este par.
Soluţie: Presupunem că a12 + a 22 + ... + a 2009
2
este impar⇒ (a1+a2+…+a2009)+
+( a + a + ... + a )=(a1+ a ) + ( a 2 + a ) + ... + ( a 2009 + a ) =a1(a1+1)+a2(a2+1)+…+
2
1
2
2
2
2009
2
1
2
2
2
2009
+a2009(a2009+1) este impar contradicţie căci fiecare termen este par, ca produs a două numere
consecutive.

G2. Ştiind că penultima sa cifră este 9 să se arate că ultima cifră a unui număr natural pătrat perfect
este 6.
Soluţie: Ultima cifră a unui pătrat perfect poate fi: 0, 1, 4, 5, 6, 9. Ţinând cont că un pătrat perfect
nu poate fi decât de una din formele 4k sau 4k+1, k∈N, obţinem că ultima cifră căutată este 6.

G3. Dacă numerele reale strict pozitive x, y, z sunt dimensiunile unui paralelipiped dreptunghic şi
verifică egalitatea: x2+y2+z2+x+y+z=6 să se arate că aria totală a acestuia nu poate fi mai mare strict
decât 6.
Soluţie: Concluzia se rescrie: 2(xy+yz+xz)≤6⇔xy+yz+xz≤3. Presupunem că xy+yz+xz>3.
Din x2+y2+z2≥xy+yz+xz⇒x2+y2+z2>3 (1).
Apoi (x+y+z)2=x2+y2+z2+2(xy+yz+xz)>3+2⋅3
=9⇒x+y+z>3 (căci x, y, z>0) (2).
Din (1)+(2)⇒x2+y2+z2+x+y+z>6 contradicţie.

G4. Dacă într-un triunghi o latură e media aritmetică a celorlalte două şi o înălţime e media
geometrică a celorlalte două să se arate că triunghiul dat are toate inaltimile congruente.
Soluţie: Cu notaţiile uzuale, să presupunem a≤b≤c (1). Cum 2S=aha=bhb=chc obţinem ha≥hb≥hc (2)
2S 2S 2S
dar şi ha = , hb = , hc = .
a b c
b+c
Din ipoteză, dacă a = ⇒ 2a=b+c.
2
Dar a≤b, a≤c⇒2a≤b+c. Deci trebuie să avem egalitate în fiecare din inegalităţile (1) adică triunghiul
a+b a+c
să fie echilateral. Analog pentru c= . Rămâne cazul b= ⇒ a+c=2b (3). Analog din (2)
2 2
2S 2S 2S 2S ( 4)
a + c 2b
rămâne de studiat cazul hb= ha ⋅ hc ⇒ = ⋅ = ⇒ b= ac ≤ = = b . Pentru a
b a c ac 2 2
(1)
avea egalitate în inegalitatea mediilor (4) trebuie să avem a=c ⇒ a = b = c .

- 19 -
CERCUL DE MATEMATICA
Tema pentru grupa de performanta la clasa a- VII -a

Operaţii cu numere iraţionale


Profesor Nedeianu Dan
Colegiul Tehnic “Domnul Tudor”

În cele ce urmează ne propunem să stabilim diverse legături între anumite numere iraţionale
sau raţionale, folosind formulele de calcul prescurtat şi diverse procedee.
De la bun început se ştie că suma, diferenţa, produsul, câtul a oricăror două numere
raţionale este tot un număr raţional (în limbaj matematic, se spune că Q este parte stabilă a lui R în
raport cu operaţiile respective).
În schimb suma (diferenţa, produsul, câtul) dintre două numere iraţionale poate fi atât număr
raţional cât şi număr iraţional. În acest sens, avem de exemplu că:
2 + ( − 2 ) = 0 ∈ Q , deşi 2 , - 2 ∉Q
( 2 + 1) − ( 2 − 1) = 2 ∈ Q , deşi 2 +1, 2 -1∉Q
( 3 − 2)( 3 + 2) = −1 ∈ Q , deşi 3 − 2, 3 + 2 ∉ Q
18 2
= 3, deşi 18, 2 ∉ Q şi apoi 2 + 3 , 2 − 3 , 2 ⋅ 3, sunt iraţionale cu
2 3
2, 3 ∉ Q .
Cu alte cuvinte, mulţimea R − Q nu este parte stabilă a lui R în raport cu operaţiile
amintite.
În continuare prezentăm o listă utilă de tipuri de probleme publicate în reviste sau date la
anumite concursuri şcolare, care se încadrează în coordonatele acestor considerente.

Exemplul 1
x
Dacă x, y ∈ R * sunt distincte cu xy , , x 3 − y 3 ∈ Q să se arate că x şi y sunt numere raţionale.
y
(Profesor Nedeianu Dan, Concursul ”Arhimede", 2006, clasa a VIII-a)
x
Soluţie: Din xy ∈ Q şi ∈ Q ⇒ x 2 ∈ Q şi y 2 ∈ Q . Apoi x( x 3 − y 3 ) = ( x 2 ) 2 − ( xy) ⋅ y 2 ∈ Q şi
y
cum x 3 − y 3 ∈ Q * , rezultă x ∈ Q şi deci y ∈ Q . Condiţia x ≠ y este esenţială, pentru că x = y = 2
verifică enunţul fără a fi numere raţionale.

Exemplul 2
Determinaţi numerele iraţionale x pentru care numerele x 2 + 2 x şi x 3 − 6 x sunt simultan numere
raţionale.
(OJM 2008, clasa a VIII-a)
Soluţie: Notând a = x + 2 x , b = x − 6 x , avem a, b ∈ Q şi b = (a − 2) x − 2a , şi cum x ∉ Q ,
2 3

deducem că a = 2 şi b = −2a , adică x 2 + 2 x − 2 = 0 , cu x = −1+ 3 .

-2020 -
CERCUL DE MATEMATICA SSM
Exemplul 3 H
Fie a, b ∈ R * , a + b ≠ 0 , astfel încât numerele a(2b 2 − a 2 ) şi b(2a 2 − b 2 ) , a 3 + b 3 sunt numere
a 2b2
raţionale. Să se arate că este număr raţional.
a+b
(Profesor Nedeianu Dan, RMT 4/2003)
Soluţie: Se observă că a(2b − a ) + b(2a − b ) = 2ab(a + b) − (a 3 + b 3 ) şi atunci ab(a + b ) ∈ Q
2 2 2 2

(1).
Însă [a(2b 2 − a 2 )]2 + [b(2a 2 − b 2 )]2 = a 6 + b 6 ∈ Q şi deci 2a 3b 3 = (a 3 + b 3 ) 2 − (a 6 + b 6 ) ∈ Q ,
a 2b2
adică a b ∈ Q (2). Din (1) şi (2) ⇒
3 3
∈Q
a+b
2
Se observă că a = 3 , b = 3 4 verifică enunţul, fără ca a şi b să fie raţionale.
16
Exemplul 4
Să se arate că dacă x ∈ R pentru care x 16 şi x 9 sunt numere raţionale, atunci x este număr
raţional.
(OLM Bucureşti 2003, clasa a VIII-a)
Soluţie: Avem 9 ⋅ 9 − 16 ⋅ 5 = 1 şi atunci (x ) , ( x 16 ) 5 ∈ Q , deci x 81 : x 80 = x ∈ Q .
9 9

Exemplul 5
Să se arate că există o infinitate de numere iraţionale x, y cu proprietatea că x + y = xy ∈ N .
(OJM 2006, clasa a VIII-a)
Soluţie: Avem ( x + y) ≥ 4 xy şi notând x + y = xy = n ⇒ n ≥ 5 .
2

1
Din y = n − x ⇒ n = x( n − x) ⇒ ( 2 x − n) 2 = n 2 − 4n , adică x = ( n ± n 2 − 4n ) ; însă pentru
2
n ≥ 5 , avem (n − 3) 2 < n 2 − 4n < (n − 2) 2 şi atunci x ∉ Q .
1 1
Astfel putem alege x = ( n + n 2 − 4 n ), y = ( n − n 2 − 4n ).
2 2

Exemplul 6
x
Se consideră x, y ∈ R * pentru care numerele , xy şi y x + ( x + 1) + x ( x + 1) sunt toate
2 2 2 2
y
numerele raţionale. Arătaţi că, de asemenea, x şi y sunt numere raţionale.
(Profesor Nedeianu Dan, Concursul „Petre Sergescu”, 2008, clasa a VIII-a)
x
Soluţie: Din ∈ Q şi xy ∈ Q , rezultă x2 ∈Q şi y2 ∈Q . Al treilea număr este
y
y( x 2 + x + 1) ∈ Q ⇒ yx 2 + y ∈ Q ⇒ y ( x 2 + 1) ∈ Q , deci y ∈ Q şi x ∈ Q .
În încheiere, vom prezenta o listă de probleme propuse, ce se pot aborda prin tehnicile etalate
anterior.
1) Fie x , y, x numere reale nenule astfel încât xy, yz, zx sunt numere raţionale.
a) Arătaţi că numărul x 2 + y 2 + z 2 este raţional.

- 21 -
CERCUL DE MATEMATICA
b) Dacă, în plus, numărul x 3 + y 3 + z 3 este raţional, să se arate că x, y , z sunt numere
raţionale.
(OJM 2001, clasa a VIII-a)
1 1 1
2) Fie a, b, c ∈ Q ∗ astfel încât + + = abc . Demonstraţi că numărul
a b c
( a 2 b 2 + 1)(a 2 c 2 + 1)(b 2 c 2 + 1) este raţional.
(Profesor Nedeianu Dan, GM 4-5/1997)
1 1 1
*Această problemă a fost propusă şi la OLM Iaşi 2003, iar pentru a = , b = , c = se
x y z
obţine problema 1 de la OJM 2002, clasa a VIII-a.
3) a) Fie x un număr real astfel încât x 2 + x şi x 3 + 2 x să fie raţionale. Arătaţi că x este număr
raţional.
b) Arătaţi că există numere iraţionale x astfel încât x 2 + x şi x 3 − 2 x să fie raţionale.
(OJM 2002, clasa a VIII-a)
4) Fie numerele reale distincte a şi b care au proprietăţile:
a 2 + b ∈ Q şi a + b 2 ∈ Q . Arătaţi că:
1+ 2 1− 2
a) Numerele a = şi b = verifică enunţul.
2 2
b) Dacă a + b ∈ Q − {1} , atunci a ∈ Q şi b ∈ Q .
a
c) Dacă ∈ Q , atunci a ∈ Q şi b ∈ Q .
b
(OJM 2004, clasa a VIII-a)
5) Fie a, b, c ∈ R * astfel ca ab, ac, bc ∈ Q . Să se arate că
(a 2 + b 2 + c 2 )(a + b + c) 2 + (ab + ac + bc) 2 este număr raţional.
(Profesor Nedeianu Dan, RMT 3/2006)
2+ n
6) Să se determine n ∈ N astfel încât ∈Q .
2 + 2007
(Concursul “Traian Lalescu” 2007, clasa a VIII-a)

BIBLIOGRAFIE

[1] Matematica în concursurile şcolare, clasele V-VIII, Editura Paralela 45, 2001-2007;
[2] Gazeta Matematică;
[3] Revista de Matematică din Timişoara.

-2222 -
CERCUL DE MATEMATICA SSM
Tema pentru grupa de performanta la clasa a- VIII -a H

PRINCIPIUL INVARIANTULUI

Prof. Elena Rimnicianu


Prof. Victor Saceanu
Scoala cu clasele I –VIII Nr. 11

In matematica “invariantul” este o marime, o relatie sau o proprietate ce ramane


neschimbata in urma aplicarii sau interventiei unei transformari.
Deci o situatie initiala este supusa in mod repetat unor transformari. De obicei se cere sa se
demonstreze ca in urma acestor transformari se poate sau nu ajunge la o anumita forma. Aceasta se
poate face alegand caracteristica obiectului care a fost supus transformarii, adica „invariantul”
transformarii. Daca in final obiectul nu poseda “invariantul” atunci el nu poate fi obtinut in urma
transformarilor descrise.

Aplicatii
1) Intr-un sistem cartezian xOy din punctul (x;y) este permisa deplasarea intr-unul din cele
patru puncte (x-2 ; y-2) ; (x-2 ; y+2) ; (x+2 ; y-2) ; (x+2 ; y+2). Demonstrati ca din punctul
(0 ;0) nu se poate ajunge prin deplasari succesive in punctul (2008 ; 2009).
Solutie
Observam ca un punct (x ;y) avand suma coordonatelor x+y un numar par se
poate deplasa intr-un punct avand suma coordonatelor tot para, acesta
fiind “invariantul ”
I (x-2 ; y-2) are suma coordonatelor x+y-4- para
II (x-2 ; y+2) are suma coordonatelor x+y- para
III (x+2 ; y-2) are suma coordonatelor x+y –para
IV (x+2 ; y+2) are suma coordonatelor x+y+4 – para
Punctul (0;0) are suma coordonatelor 0, deci numar par, el se poate deplasa succesiv in puncte cu
aceeasi paritate a sumei coordonatelor, deci nu se poate deplasa in punctul (2008;2009) care are
suma coordonatelor impara.

2) Pe o tabla scriem numerele de la 1 la 2007. Printr-o operatie intelegem ca stergem doua


numere oarecare de pe tabla si in locul lor scriem modulul diferentei celor doua numere. Sa
se demonstreze ca dupa 2006 operatii pe tabla ramane un numar par.
Solutie
La orice pas i, inainte de stergerea a doua numere a si b, fie Si+1 suma numerelor de pe tabla si Si
suma numerelor de pe tabla dupa stergerea numerelor a si b. Atunci avem “invariantul ”:
⎧2a, daca a < b
Si+1- Si = a + b - ⏐a-b⏐= ⎨ . Deci Si+1- Si este totdeauna un numar par.
⎩2b, daca a ≥ b
Suma 1+2+3+…+2007= 1004⋅2007 este un numar par, rezulta ca S1 este tot un numar par. Dar S1
este ultimul numar de pe tabla, deci pe tabla a ramas un numar par.

- 23 -
CERCUL DE MATEMATICA
3) Trei elevi sunt asezati in linie dreapta in ordinea X, Y, Z. Ei incep sa sara capra, adica unul
peste altul (dar nu peste doi odata). Pot fi in aceasta ordine dupa 2009 sarituri ?
Solutie
In urma unei sarituri, numarul elevilor inversati creste sau se micsoreaza cu 1, acesta fiind si
“invariantul ”. Dupa un numar de 2009 sarituri (numar impar) va exista un numar impar de perechi
de elevi inversati. Cum ordinea initiala nu contine o astfel de pereche, rezulta ca nu este posibil.

4) Consideram numerele a ; b + a ; b - a , a ≥ 0. Dupa un pas inlocuim fiecare din numerele


date prin media aritmetica a celorlalte doua. Se pot efectua mai multi pasi astfel incat sa
obtinem numerele: a –1; b + 1+ a ; b+1 - a .
Solutie
In acest caz ramane ca “invariant ” suma numerelor. Daca x, z,y sunt numerele, atunci ele se
y+z z+x x+ y
inlocuiesc cu ; ; care au suma:
2 2 2
x + y x + z y + z 2( x + y + z )
+ + = = x+y+z
2 2 2 2
Suma numerelor initiale este: a+b+ a +b- a = a+2b, iar suma numerelor finale este: a-
1+b+1+ a +b+1- a = a +2b +1. Deci plecand de la numerele: a; b + a ; b - a nu se poate
ajunge la numerele a-1 ; b + 1+ a ; b+1 - a .

BIBLIOGRAFIE :
V. Pop, V. Lupsor – Matematica pentru grupele de performanta, clasele a VI-a, a VII-a.

-2424 -
CERCUL DE MATEMATICA SSM
Tema pentru grupa de performanta la clasa a- IX -a H
Invarianţi
prof. Draga Tătucu Mariana
elev Nistor Adriana Mihaela
Colegiul Naţional „Gheorghe Ţiţeica”

Se întâmplă ca să întâlnim de multe ori probleme de combinatorică în care


avem o configuraţie iniţială, asupra căreia putem efectua un şir de transformări.
Se pune întrebarea dacă putem ajunge la o altă configuraţie dată cu ajutorul
acestor transforări. În astfel de probleme este utilă noţiunea de invariant, o
caracteristică care nu se schimbă în urma operaţiilor permise. Dacă această
caracteristică este diferită pentru configuraţia iniţială şi cea finală, răspunsul
este negativ.
Spre exemplu: Pe tablă sunt scrise trei numere. La fiecare pas putem
aduna două din ele, să scădem din sumă al treilea număr, şi să scriem rezultatul
obţinut în locul celui de-al treilea număr. Dacă pe tablă erau 1,2,3 putem oare
să obţinem 10,20,30?
Operaţia constă în transformarea tripletului (a,b,c) în (a,b,a+b-c). Trebuie deci să
căutăm o expresie (simetrică) în a,b,c astfel încât ea să nu se schimbe în urma
acestei operaţii, adică f(a,b,c)= f(a,b,a+b-c). Cunoaştem astfel de funcţii:
trinoamele de gradul doi. În cazul dat, dacă f(a,b,x)= x(a+b-x)+E, atunci f(a,b,c)=
f(a,b,a+b-c). Deci f(a,b,c)= c(a,b,a+b-c) +E, unde E nu depinde de c.Acum găsim
uşor că f(a,b,c)= ab+bc+ca-a2 – b2 –c2. această expresie este invariantă în urma
operaţiei noastre. Cum f(1,2,3) = -3, iar f(10,20,30)= -300, problema este
rezolvată.
Alt tip de invarianţi pe mulţimi constă în colorarea unor elemente şi
sumarea unor cantităţi pentru anumite culori. De exemplu dacă efectuăm
operaţii pe o tablă de şah, putem considera suma unor cantităţă doar pe
câmpurile negre ale tablei. Spre exemplu: În vârfurile unei table 3x6 sunt plasate
numerele de la dela 1 la 18. Este permis să alegem două câmpuri vecine pe
orizontală sau diagonală şi să micşorăm cu 1 numerele din ambele câmpuri.
Putem oare prin intermediul acestor operaţii să înlocuim numerele din fiecare
câmp cu opusele lor (adică x cu -x)?
În această problemă colorăm tabla alb şi negru ca la şah. Considerăm D
diferenţa dintre suma numerelor de pe câmpurile albe şi suma numererlor de pe
câmpurile negre. Deoarece o pereche de câmpuri vecine conţine un câmp alb şi
unul negru, D este invariantă . Dacă am putea inlocui fiecare număr prin opusul
său, diferenţa ar deveni –D, deci acest lucru este posibil numai pentru D= -D,
adică D= 0. Însă atunci suma numerelorde pe toată tabla ar fi pară, iar ea este
iniţial 1+2+3+4+........+18= 171 şi este impară. Vom expune câteva probleme care
se rezolvă cu ajutorul invarianţilor:
Exemplul 1:
Pe fiecare câmp al tablei de şah 8x8 avem câte un beculeţ. Iniţial toate
becurile sunt aprinse.Putem lua două câmpuri adiacente diagonal şi să
schimbăm starea lor. Este posibil ca după câteva operaţii toate becurile să fie
stinse, cu excepţia becurilor de pe câmpurile A1, A2?
Indicaţie: Câmpurile adiacente diagonal au aceeaşi culoare. Deci paritatea
numărului de becuri aprinse de culoare albă este invariantă.

- 25 -
CERCUL DE MATEMATICA
Exemplul 2:
Iniţal avem scris pe tablă numărul 4. La ficare pas, putem fie să ridicăm
numărul de pe tablă la pătrat, sau să-l micşorăm cu 3. Putem oare avea scris
numărul 2007 după câteva operaţii?
Indicaţie: Restul numărului prin impărţirea la 3 este intotdeauna 1.
Exemplul 3:
În vârfurile unui poligon regulat sunt scrise câteva numere. Iniţial doar
unul din ele este nenul. Putem alege câteva dintre vârfurile poligonului mare care
să formeze un poligon regulat mai mic şi să adăugăm 1 la fiecare din numerele
din aceste vârfuri. Este oare posibil ca după câteva operaţii toate numerele să
devină egale?
Indicaţie: Dacă A1, A2, A3, .........., An sunt vârfurile poligonului, iar k1, k2, ......,
kn
sunt numerele scrise în ele, vectorul k1OA1 + k2OA2 +..........+ knOAn este
invariant, unde O este centrul poligonului.
Exemplul 4:
Pe tablă sunt scrise trei numere. Putem efectua următoarea operaţie:
alegem unul din ele c şi-l înlocuim cu 2a + 2b –c, dacă a şi b sunt celelalte
numere. Este oare posibil să obţinem (5,13,42) din (1, 21, 42)?
Indicaţie: Pentru această problemă invariantă este funcţia g(a,b,c)= 2ab+2bc+2ca
– a2-b2-c2.
Probleme propuse:

Problema 1: Fişele de la casino pot fi albe sau negre. Într-un rând sunt 10
fişe albe. La un pas putem scoate o fişă albă de pe tablă şi să schimbăm culoarea
fişelor vecine cu ea. Rămâne o singură fişă. Ce culoare are ea?
Problema 2: Fie o tablă de şah 10x10. Pe câmpurile diagonalei principale se
află câte un pion. Avem voie să alegem doi spioni şi să-i mutăm cu câte un câmp
în sus. Putem oare să aducem cu astfel de opraţii toţi pionii pe o linie?
Problema 3: Pe o tablă de şah sunt scrise numerele 1,2,3,...............,2001.
Putem înlocui două numere a şi b cu ab/(a+b+1). Repetăm operaţia aceasta până
pe tablă rămâne un singur număr. Care este acesta?
Bibliografie:
„Olimpiadele de matematică” - Iurie Boreico, Tudor Micu, Andrei Ciulpan,
Laszlo Szilard - Ed.GIL2006

-2626 -
CERCUL DE MATEMATICA SSM
Tema pentru grupa de performanta la clasa a-X-a H

Câteva probleme generate de


exponenţiala de bază 2
Prof. Chirfot Carmen – Victoriţa
Colegiul Tehnic “Domnul Tudor”

2 3
.... ....
2 2
2 2
Problema 1: Să se calculeze [ 1242 4 ], respectiv [ 124243 ]
4 43 n ∈ N * , unde [x ] reprezintă
2 apare de n ori 2 apare de n ori

partea întreagă a lui x.


Soluţia 1:
1 < 2 < 2 . Aplicând funcţia exponenţială de bază 2 ( este strict crescătoare ), obţinem
2 2
1 2 2 2 exp de baza 2 2 2 2 2 2
2 < 2 < 2 ⇒ 2< 2 <2 ⇒ 2 < 2 < 2 ⇒ 2 < 2 <2.
2 2
.... ....
2 2
2 2
Prin inducţie, se demonstrează că 1 < 124243 < 2 ⇒ [ 124243 ] = 1 . În mod analog,
2 apare de n ori 2 apare de n ori

3
....
2
2
demonstrăm că [ 1242 4 ]=1.
4 43
2 apare de n ori

Soluţia 2:
2 3 4 2 3
.... .... .... .... ....
2 2 2 2 2
2 2 2 2 2
1 < 1242 4 < 1242
4 43 4 < 1242
4 43 4 = 2 ⇒ [ 1242
4 43 4 ] = [ 1242
4 43 4 ] =1.
4 43
2 apare de n ori 2 apare de n −1 ori 2 apare de n −1 ori 2 apare de n ori 2 apare de n −1 ori

2
....
2
2
Problema 2: Să se calculeze [ 1242 4 ⋅ ln 2 ],
4 43 n∈ N* .
2 apare de n ori

Soluţie:
2 2 2
.... .... ....
2 2 2
2 2 2
Deoarece 1 < 1242 4 < 2 ⇒ ln 1 < ln ( 1242
4 43 4 ) < ln 2 ⇒ 0 < 1242
4 43 4 ⋅ ln 2 < ln 2 < ln e = 1 .
4 43
2 apare de n +1 ori 2 apare de n +1 ori 2 apare de n ori

2
....
2
2
Prin urmare, [ 1242 4 ⋅ ln 2 ] = 0 .
4 43
2 apare de n ori

2
2 2
.... ....
2 2
2 2
Problema 3: Să se demonstreze că 12442443 − 1242 4 < 1,
4 43 n∈N*.
2 apare de n +1 ori 2 apare de n ori

- 27 -
CERCUL DE MATEMATICA
Soluţie:
2 2
2 2 2 2
.... .... .... ....
2 2 2 2
2 2 2 2
Deoarece 1 < 1242 4 < 1244244
4 43 3 < 2 ⇒ 1244244
3 − 1242 4 < 2 − 1 = 1 ⇒ q.e.d .
4 43
2 apare de n ori 2 apare de n +1 ori 2 apare de n +1 ori 2 apare de n ori

2
....
2
2
Problema 4: Să se calculeze lim 1242 4 .
4 43
n→ ∞
2 apare de n ori

2
....
2

4 ⇒ ln a n +1 = a n ln 2 . Evident, (a n ) n ≥1 este strict crescător şi


2
Soluţie: Notăm a n+1 = 1242
4 43
2 apare de n +1 ori

mărginit. Conform teoremei lui Weierstress, şirul are limită finită. (a n )n≥1 este mărginit deoarece
2 2
1 2 2 2 exp de baza 2 2 2 2 2 2
1< 2 < 2 ⇒ 2 < 2 < 2 ⇒ 2< 2 <2 ⇒ 2 < 2 < 2 ⇒ 2 < 2 <2.
2
....
2
2
Prin inducţie, procedând analog, se demonstrează că 1 < 1242 4 < 2 ⇒ a n ∈ (1, 2) .
4 43
2 apare de n ori

Prin trecere la limită în relaţia de recurenţă, găsim ln l = l ln 2 . Pentru rezolvarea ecuaţiei,


1 1 1
considerăm f : [1, 2] → R, f ( x) = ln x − x ln 2 ⇒ f ' ( x) = − ln 2 . Din f ' ( x) = 0 ⇒ = ln 2 ⇒ x = .
x x ln 2

Deoarece, 2 <
1

1
∉ [1, 2] . Cum f’ x 1 2
ln 2 ln 2
f’(x) + + + + + + + + + + + + + + +
strict pozitivă, avem f strict crescătoare.
f(x) f(2) = 0
Facând tabelul de semn, se observă că
unica rădăcină a funcţiei f inclusă în intervalul [1, 2] este x = 2 ⇒ l = 2 .

Probleme propuse:
1. Să se demonstreze că ε > 0 ∃ n 0 ∈ N * astfel încât n ≥ n0 , n∈N* să
2
2 2
.... ....
2 2
2 2
avem 12442443 − 1242 4 <ε .
4 43
2 apare de n +1 ori 2 apare de n ori

2
....
2
2
2. Să se calculeze lim n( 1242 4 − 2) .
4 43 n→∞
2 apare de n ori

-2828 -
CERCUL DE MATEMATICA SSM
Tema pentru grupa de performanta la clasa a- XII -a H

Centralizatorul unui element într-un grup.


Centrul unui grup. Teorema lui Lagrange.
Indicele unui subgrup într-un grup.

Prof.Bondoc Gabriela Roxana


Grup Scolar Industrial Auto
Definiţia 1. Pentru x∈G vom nota
CG(x) = { y∈G : xy=yx } care se numeşte centralizatorul lui x în G si Z(G)=
I CG(x) care se numeşte centrul lui G.
x∈G

In mod evident Z(G)= { x∈G ; xy = yx, pentru orice y∈G }.

Propoziţia 2. Pentru orice x∈G, CG(x)≤G.


Demonstraţie. Dacă y, z ∈CG(x), atunci yx=xy şi zx=xz. Deducem că y-1 x=xy-1 iar (y-1z)x=y-1(zx)
= y-1(xz) = (y-1 x)z = = (xy-1)z = x(y-1z), adică y-1z ∈ CG(x), deci CG(x) ≤G.
Corolar 3. Z(G)≤G.
Demonstraţie. Avem Z(G)= I CG(x) şi Z(G)≤G.
x∈G

Fie acum H≤G şi x∈G.


Definim xH = { xh : h∈H } şi Hx = {hx : h∈H }. Mulţimea xH (Hx) poartă numele de
clasa la stânga (dreapta) a lui x în raport cu H.
Propoziţia 4. Dacă x, y∈G şi H≤G atunci
(i) xH = yH ⇔ x -1y∈H
(ii) Hx = Hy ⇔ xy-1∈H.
Demonstraţie. (i). Să presupunem că xH = yH. Cum 1∈H , x=x ⋅1 ∈ xH = yH, adică x = yh cu h
∈ H. Deducem că y-1x=h ∈H şi cum h-1 ∈H avem că h-1 = x-1 y ∈ H. Reciproc, fie x-1 y=h∈H şi
z∈xH, adică z=xk cu k∈H. Cum x=yh-1 avem z=(yh-1)k = y(h-1 k) , adică z∈yH (căci h-1 k∈H ),
deci xH⊆yH . Analog deducem că şi yH⊆ xH, de unde xH=yH.
(ii). Ca şi în cazul (i).
Corolar 5. Dacă H≤G, atunci pentru x∈G, xH = H (sau Hx = H) ⇔ x∈H. În
particular, 1⋅H = H.
Vom nota (G/H)s = {xH : x∈G} şi (G/H)d = {Hx : x∈G}
Propoziţia 6. (G/H)s şi (G/H)d sunt partiţii ale lui G.
Demonstraţie. Este suficient să probăm pentru (G/H)s. Deoarece pentru orice x∈G avem x=x⋅1 ∈
xH deducem că U xH = G.
x∈G

Fie acum x, y∈G şi să demonstrăm că xH=yH sau xH∩yH=Ø . Avem că x-1 y∈H sau x-
1
y∉H. Dacă x-1 y ∈H, conform Propoziţiei 4, xH=yH.
Să presupunem acum că x-1 y∉H. Dacă ar exista z∈xH∩yH, atunci z=xh=yk cu h, k ∈H şi
am deduce imediat că x-1 y= hk-1 ∈H -absurd. Deci în cazul x-1 y∉H avem xH∩yH=Ø.
Propoziţia 7. Funcţia f : (G/H)s →(G/H)d , f(xH)=Hx-1 pentru orice x∈G este o bijecţie.
Demonstraţie. Pentru x, y∈G echivalenţele xH=yH ⇔ x-1 y ∈ H ⇔ x-1(y-1)-1 ∈ H ⇔ Hx-1= Hy-1
(conform Propoziţiei 4) ne arată că f este corect definită şi că este injectivă. Cum surjectivitatea
lui f este imediată, deducem că f este bijecţie.

- 29 -
CERCUL DE MATEMATICA
Din propoziţia precedentă deducem că |(G/H)s|= |(G/H)d|; acest număr cardinal se
notează |G:H| şi poartă numele de indicele lui H în G.
Lema 8. Dacă H≤G şi x∈G, atunci |xH|=|Hx|=|H| .
Demonstraţie. Este suficient să arătăm că mulţimile xH şi H sunt echipotente iar în acest sens
definim fx : H →xH, fx(h) = xh pentru orice h∈H.
Dacă h, k∈H şi fx(h) = fx(k) atunci xh=xk deci h=k adică f este injectivă. Cum fx este în
mod evident şi surjectivă, deducem că fx este o bijecţie şi astfel |xH|=|H|.
Teorema 9. Dacă H≤G, atunci
|G|= |H|⋅|G:H|.
Demonstraţie. Cum (G/H)s este o partiţie a lui G avem |G| = ∑ |xH|(sumarea făcându-se după
x∈G

clase distincte). Ţinând cont de Lema 8. deducem că |G|=|H|⋅|G:H|.


În cazul în care G este un grup finit, atunci |G|, |H| şi |G : H| sunt numere naturale iar
relaţia |G|=|H|⋅|G:H|arată că |H| este un divizor al lui |G|.
Teorema 10.(Lagrange) Ordinul oricărui subgrup al unui grup finit divide ordinul
grupului.
Corolar 11. Dacă G este un grup finit de ordin n, atunci x n =1 pentru orice x ∈ G.
Demonstraţie. Dacă k=o(x), atunci xk =1 şi k|n (conform teoremei lui Lagrange), adică n=kt cu
t∈ℕ. Atunci xn=xkt =(xk)t=1 t=1.

Aplicatii

1.Fie G un grup comutativ de ordin n. Arătaţi că produsul celor n elemente ale


lui G este egal cu produsul elementelor de ordin cel mult 2. Aplicând acest rezultat
grupului multiplicativ ( Z *p , ⋅) , p prim să se demonstreze că p ( p − 1)!+1 .
Rezolvare: Avem ∏ x = ( ∏ x) ⋅ ∏ x şi vom demonstra că ( ∏ x) = 1 (*).
x∈G x∈G x∈G x∈G
o ( x )> 2 o ( x )≤2 o ( x )>2

Dacă x ∈ G cu o(x)>2 atunci o( x ) = o ( x −1 ) > 2 .Pe de altă parte x ≠ x −1 (altfel x2=1).


Deci în produsul (*) dacă apare x apare s inversul său deci ( ∏ x ) = 1 , de unde concluzia.
x∈G
o ( x )>2
Pentru a demonstra cea de a doua afirmaţie cunoscută sub numele de teorema lui Wilson, se ţine

cont de faptul că singurele elemente din Z *p de ordin mai mic sau egal cu 2 sunt 1̂ şi p − 1 , deci
^ ∧ ^ ^
conform celor de mai înainte 1̂ ⋅ 2̂ ⋅ ... ⋅ p − 1 = p − 1 ⇒ ( p − 1)! = p − 1 ⇒ p ( p − 1)!+1 .
2. Fie n ≥ 2 un număr natural. Să se arate că n este un număr natural prim daca
şi numai dacă orice grup cu n elemente are exact două subgrupuri.
Rezolvare:Dacă n este un număr prim , considerând un grup arbitrar cu n elemente, orice subgrup al
acestuia va avea ca ordin un divizor al lui n (Teorema Lagrange), adică pe 1 sau n. Rezultă că G are
numai două subgrupuri şi anume {e} şi G.
Reciproc, să admitem că orice grup cu n elemente are numai două subgrupuri şi să arătăm
că n prim. Presupunem prin reducere la absurd că n nu este prim şi fie atunci un divizor prim al său.
^
Să considerăm grupul aditiv cu n elemente Z n.. Subgrupul ciclic H generat de clasa n / p are p
elemente şi cum 1<p<n rezultă H este subgrup al lui (Zn,+).Deci am găsit un grup cu n elemente
care are trei subgrupuri , contradicţie cu ipoteza.

-3030 -
CERCUL DE MATEMATICA SSM
3. Fie G un grup şi x un element de ordin finit din G. Dacă m şi n sunt doiH
întregi pozitivi cu proprietăţile: (n,m)=1, ord(xn)=m, ord(xm)=n, atunci ord(x)=nm.
Rezolvare: Fie k=ord(x) Cum ord(xn)=m rezultă (xn)m =e adică xnm=e. De aici rezultă că nmM k . (1)
Cum xk=e, putem scrie xkm=e, adică (xm)k =e şi cum ord(xm)=n,rezulta k M n. (2) Analog se arată că
k M m .(3) Dar (n,m)=1 rezultă k M n.m (4)
Din relaţiile (1) şi (4) rezultă k=mn.
4.Fie G un grup iar K ≤ G .Să se demonstreze că CG ( K ) = 1 ⇔ Z ( H ) = 1, ( ) H a.î.
K ≤ H ≤G.
Rezolvare:Dacă alegem H a.î K ≤ H ≤ G , deoarece Z(H) ≤ CG(K)=1 ⇒ Z(H)=1.
Reciproc, presupunem că Z(H)=1 pentru orice H a.î. K ≤ H ≤ G .
Dacă x ∈ CG(K) , atunci pentru H= K ∪ {x} avem K ≤ H ≤ G şi x ∈ Z(H)=1, deci CG(K)=1.
5.Să se demonstreze că pentru n ≥ 3 ,Dn are un singur subgrup ciclic de ordin n.
Rezolvare:Fie H= ρ ≤ Dn ; deoarece o( ρ ) = n deducem că H este un subgrup ciclic de ordin n.
Orice element din Dn\H este se forma ρ k ε , k=0,1,…,n-1.De asemenea avem că
ερ = ρ n −1ε = ρ −1ε .Dacă presupunem că ερ k = ρ −k ε , atunci
ερ k +1 = ερρ k = ρ −1 ρ −k ε = ρ −( k +1)ε prin urmare ερ k = ρ − k pentru orice k=0,1,...,n-1 şi
( ρ k ε ) 2 = ρ k ερ k ε = ρ k ρ − k ε 2 = 1 .
Rezultă că pentru orice x ∈ D n \ H avem o(x)=2. Dacă H ′ = x , x ∈ Dn este un subgrup
ciclic de ordinul n, atunci o(x)=n>2, deci x ∈ H .
Dar atunci H ′ ≤ H , H ′ = H = n , deci H ′ = H .

BIBLIOGRAFIE
1. D. Buşneag : Teoria Grupurilor, Ed. Universitaria, Craiova, 1994
2. D.Buşneag : Capitole speciale de algebră,Ed. Universitaria,Craiova, 1997.
3. D. Buşneag, Fl. Chirteş, D. Piciu : Algebra, Ed. Universitaria, Craiova, 2001.
4. Al. Dincă : Lecţii de algebră, Ed. Universitaria, Craiova, 2000.
5. I. D. Ion, N. Radu : Algebra, Ed. Didactică şi Pedagogică, Bucureşti, 1991.
6. I. D. Ion, C. Năstăsescu, C. Niţă : Complemente de Algebră, Ed. Ştiinţifică şi Enciclopedică, Bucureşti,
1994.
7. D. Popescu, C. Vraciu : Elemente de teoria grupurilor finite, Ed. Ştiinţifică şi Enciclopedică, Bucureşti,
1986.

- 31 -
CERCUL DE MATEMATICA
Tema pentru grupa de performanta la clasa a- XII -a

ASUPRA ELEMENTELOR SIMETRIZABILE


DINTR-UN MONOID
Prof.dr.Cainiceanu Gheorghe
Prof.Leonard Giugiuc CNT

§1.Rezumat
Tema se ocupa de prezentarea unor aspecte des folosite in aplicatii referitoare la functii injective si
surjective ,introducerea notiunilor de elemente inversabile la stanga sau la dreapta intr-un monoid,
rezultate de baza referitoare la acestea, si o lista de probleme rezolvate,respective propuse.

§2.Cateva rezultate generale


Vom nota in cele ce urmeaza cu A o multime infinita si cu FA multimea tuturor functiilor definite
pe A cu valori in A.
Se presupun cunoscute notiunile de functie injectiva, surjectiva, bijectiva, inversabila.
TEOREMA 1 Fie f,g : A→ A .Atunci avem:
1) Daca f si g sunt injective,atunci f o g este injectiva.
2) Daca f si g sunt surjective,atunci f o g este surjectiva.
3) Daca f si g sunt bijective,atunci f o g este bijectiva.
Demonstratie. 1) Fie x≠y. Deoarece functia g este injectiva avem g(x)≠g(y).Dar acum invocam
injectivitatea lui f si deducem f(g(x))≠f(g(y)) adica ( f o g )(x) ≠( f o g )(y).
2) Pentru un z din A privit codomeniu pentru f exista y din A codomeniu pentru g astfel incat
f ( y ) = z .Acum exista x din A,domeniul lui g pentru care g ( x) = y .
In concluzie, z ∈ A∃x ∈ Aa.i. f ( g ( x)) = z.
3)Este o consecinta pentru 1) si 2).
TEOREMA 2 O functie f ∈ FA este inversabila daca si numai daca este bijectiva.
TEOREMA 3 Daca pentru f , g ∈ FA avem f o g bijectiva, atunci g este injectiva iar f surjectiva.
Demonstratie. Vom demonstra ca g este injectiva.
Daca x, y ∈ A, x ≠ y ⇒ ( f o g )( x) ≠ ( f o g )( y ) ⇒ f ( g ( x)) ≠ f ( g ( y)) ⇒ g ( x) ≠ g ( y )
Vom demonstra ca f este surjectiva. Fie z ∈ A .Atunci ∃x ∈ A( f o g )( x ) = z ⇒ f ( g ( x)) = z.
Notam g ( x) = y ⇒ f ( y ) = z .Am aratat ca: z ∈ A∃y ∈ A | f ( y ) = z ,deci f este surjectiva.
TEOREMA 4 Fie f ∈ FA o functie injectiva si nesurjectiva. Atunci exista g ∈ FA o functie
neinjectiva si surjectiva astfel incat g o f = 1A .
Demonstratie. Daca f este injectiva, functia f1 : A → Im f , f 1 ( x) = f ( x) este o bijectie.deci are
⎧ f −1 ( x ) daca, x ∈ Im f
inverse f1−1 : Im f → A .Definim functia g : A → A, g ( x) = ⎨ 1
⎩ a daca, x ∈ A \ Im f
Unde a este un numar arbitrar fixat din A.Evident sunt adevarate urmatoarele
x ∈ A, g ( f ( x)) = g ( f1 ( x)) = f 1−1 ( f ( x)) = x
g este surjectiva (imaginea este egala cu codomeniul).
Desigur g nu este injectiva deoarece ∃x ∈ Im f , ∃y ∈ A \ Im f , a.i.g ( x) = g ( y ) = a .
TEOREMA 5 Fie g ∈ FA o functie neinjectiva si surjectiva. Atunci exista o functie f ∈ F A
injectiva si nesurjectiva astfel incat g o f = 1A .
Demonstartie. Pentru orice x∈A avem g −1 ({x}) ≠ Φ ,pentru ca g este surjectiva.

-3232 -
CERCUL DE MATEMATICA SSM
Cum g este neinjectiva exista cel putin un x∈A astfel incat multimea g −1
({x}) are ce putin douaH
elemente. Pentru fiecare x∈A alegem un unic yx∈ g −1 ({x}) .
Notam B1= U { y x } . Evident, B1 ⊂ B si incluziunea este stricta,iar g 1 : B1 → A, g1 ( y x ) = g ( y x ) este
x∈A

o bijectie.Deci exista inverse sa. Fie acum x ∈ A ⇒ ( g o g1−1 )( x) = g ( g1−1 ( x)) = g ( y x ) = x .


In concluzie, g 1−1 este functia cautata.
⎧− x x∈Q

APLICATIE Fie f : R → R, f ( x) = ⎨ 1 .Sa se arate ca f este bijectiva si sa se
x∈R \Q
⎪⎩ x
determine inverse sa.
⎧ − f ( x) , f ( x) ∈ Q ⎧ − f ( x), x ∈ Q x∈Q
⎪ ⎪ ⎧ x,
Solutie: ( f o f )( x) = f ( f ( x)) = ⎨ 1 =⎨ 1 =⎨ = 1 ( x)
, f ( x) ∈ R \ Q , x ∈ R \ Q ⎩ x, x ∈ R \ Q R
⎪⎩ f ( x) ⎪⎩ f ( x)
Cum 1R bijectiva deducem ca f este injective si surjectiva (TEOREMA 3). Deci f este inversabila si
prin compunere cu inversa deducem f −1 = f .

§3Elemente simetrizabile stanga,dreapta


In monoidul ( F ,o) vom considera functia f surjectiva si neinjectiva.
TEOREMA 6 Functia f este simetrizabila la dreapta, dar nu este simetrizabila la stanga.
Demonstratie. Din TEOREMA 5 ,avem ca exista o functie g : A → A ,injective si nesurjectiva cu
proprietatea ca f o g = 1R ,deci f este inversabila la dreapta. Presupunand prin absurd ca f ar fi
simetrizabila si la stanga am contrazice Teorema 3.
TEOREMA 7 Daca f este o functie injective si nesurjectiva atunci in monoidul ( F ,o) ,ea este un
element simetrizabil la stanga si nesimetrizabil la dreapta.
Demonstratie. Din Teorema 4 deducem ca exista g : A → A surjectiva si neinjectiva cu
proprietatea g o f = 1A deci f e simetrizabila la stanga. Daca f ar fi simetrizabila la dreapta,din
Teorema 3 avem ca f este surjectiva,contradictie.
COMENTARIU.
Considerentele de mai sus arata necesitatea imperioasa ca atunci cand determinam elemental
simetric al unui element dintr-o structura sa verificam x ∗ y = y ∗ x = e .
§4 Probleme de rezolvat
1. Consideram A, B ∈ M n ( Z ), a.i. A, A + B, A + 2 B,... A + nB, inversabile in M n (Z ) .
Demonstrati ca A + (2k − 1) B este inversabila in M n (Z ) .

2. Se considera inelul (A,+,·), cu elemental unitate 1.Aratati ca daca 1+xy este


inversabil,atunci si 1+yx este inversabil, unde x,y, sunt elemente fixate in inel.

3. Fie A, B ∈ M n ( R), cu, A + B = AB .Demonstrati ca:


a) ( I n − A) −1 = I n − B b) AB = BA .

4. Pe multimea M se considera o lege decompozitie asociativa notata multiplicativ. Se


presupune ca exista a∈M cu proprietatea ca y ∈ M , y ∈ ama = {axa | x ∈ M }
Sa se arate ca legea de compozitie admite un element neutru.

Bibliografie
[1]C.Nastasescu,C.Joita,M.Brandiburu,D.Joita - Culegere de exercitii de algebra pentru liceu, Bucuresti,1996
[2]I.D.Ion,A.Ghioca,N.Nedita - Matematica,clasa a XII-a,EDP 1983
[3]C.Nastasescu,C.Nita,C.Vraciu - Bazele Algebrei,VolI, Editura Academiei Romane-Bucuresti’86

- 33 -
CERCUL DE MATEMATICA
Tema pentru grupa de performanta la clasa a- XII -a

Extinderi de inele şi corpuri (I).

prof. Adrian Lupu, Dr. Tr. Severin

În cele ce urmează ne propunem să trecem în revistă câteva elemente legate de extinderile


de inele şi corpuri, prin câteva grupe de exemple considerate relevante, unele completate şi cu mici
suplimente teoretice, accesibile elevilor de clasa a XII-a şi foarte utile şi în alte aplicaţii.
Facem precizarea că vom viza noţiunea de inel introdusă mai general, în sensul că nu mai
impunem condiţia ca şi legea a doua să aibă element neutru. Dacă acest lucru s-ar întâmpla am avea
cazul particular al inelului unitar, din manualul de algebră antedecembrist. Această manieră de
abordare va apărea oricum la nivel universitar, dar este preluată şi în unele din recentele manuale
alternative sau culegeri de liceu (de exemplu cel de Mircea Ganga). Restul noţiunilor legate de
substructurile şi extinderile de structuri din titlu se introduce similar.
Pentru ca elevii să conştientizeze ce diferenţe pot apărea din această abordare propunem
pentru început parcurgerea următoarelor exerciţii:
a) Să se arate că
⎡1 ⎤ ⎧ a ⎫
Z ⎢ ⎥ = ⎨ n , a ∈ Z, n ∈ N ⎬ ⊂ Q
⎣ 2⎦ ⎩2 ⎭
este subinel al lui (Q,+,⋅)
b) Arătaţi că singurul subinel unitar al lui Z este Z însuşi (via subgrupurile care sunt de
forma nZ şi, fiind unitar, 1 ∈ Z ⇒ n = 1)
⎡1⎤ 1
Obs. Z ⎢ ⎥ este cel mai mic subinel al lui Q care conţine Z şi .
⎣2⎦ 2
Generalizare: Dacă p este un număr prim atunci
⎡1⎤ ⎧ a ⎫
Z ⎢ ⎥ = ⎨ n , a ∈ Z, n ∈ N ⎬ ⊂ Q este subinel al lui (Q,+,⋅) .
⎣ p⎦ ⎩ p ⎭
c) Z [ 2 ] = {a + b
3 3
}
2 + c3 4 | a, b, c ∈ Z , cu adunarea şi înmulţirea obişnuite , este subinel în
(R,+,⋅) .
⎧ ⎛a x⎞ ⎫
d) A = ⎨M a, x = ⎜⎜ ⎟⎟ | a ∈ Q, x ∈ R ⎬ este subinel comutativ şi unitar în raport cu adunarea şi
⎩ ⎝0 a⎠ ⎭
înmulţirea matricilor din M2 (R ) .
e) Arătaţi că următoarele mulţimi de numere, împreună cu operaţiile obişnuite de adunare şi
înmulţire sunt subinele ale lui (C ,+,⋅) , precizându-le pe cele unitare.
1) 2Z (numerele întregi, pare) 2)mZ , unde m ∈ Z (deci multiplii întregi ai lui m)
{ } {
3) a + b 3 | a , b ∈ Z 4) a + b 3 | a, b ∈ 2Z }
{ }
5) a + b 3 | a, b ∈ Q 6) {a + bi | a, b ∈ Q} - inelul întregilor lui Gauss
7) {a + bi | a, b ∈ 3Z} 8) {a + bi | a, b ∈ Q}

{
9) a + b 3i | a, b ∈ Z } ⎧ a + bi 3
10) ⎨
2

| a , b ∈ Z,, a si b au aceeasi paritate⎬
⎩ ⎭
{
11) x ∈ R , x = a + b 2 + c 3 + d 6 , a, b, c, d ∈ Q }
f) Arătaţi că următoarele mulţimi de matrice formează subinele ale inelelor de matrice
pătratice corespunzătoare, împreună cu operaţiile uzuale de adunare şi înmulţire, precizând care
dintre aceste subinele sunt unitare şi, respectiv, care sunt comutative:

-3434 -
CERCUL DE MATEMATICA SSM
⎧⎛ a 3b ⎞ ⎫ ⎧⎛ a 3b ⎞ ⎫ H
1) ⎨⎜⎜ ⎟⎟, a, b ∈ Z⎬ 2) ⎨⎜⎜ ⎟⎟, a , b ∈ 2 Z ⎬
⎩⎝ b a ⎠ ⎭ ⎩⎝ b a ⎠ ⎭
⎧⎛ a 3b ⎞ ⎫ ⎧⎛ a − b ⎞ ⎫
3) ⎨⎜⎜ ⎟⎟, a, b ∈ Q ⎬ 4) ⎨⎜⎜ ⎟⎟, a, b ∈ Z ⎬
⎩⎝ b a ⎠ ⎭ ⎩⎝ b a ⎠ ⎭
⎧⎛ a − b ⎞ ⎫ ⎧⎛ a − b ⎞ ⎫
5) ⎨⎜⎜ ⎟⎟, a, b ∈ 3Z ⎬ 6) ⎨⎜⎜ ⎟⎟, a, b ∈ Q ⎬
⎩⎝ b a ⎠ ⎭ ⎩⎝ b a ⎠ ⎭
⎧⎛ a b⎞ ⎫ ⎧⎛ a 0⎞ ⎫
7) ⎨⎜⎜ ⎟⎟ | a, b ∈ Z ⎬ 8) ⎨⎜⎜ ⎟⎟ | a, b ∈ Z ⎬
⎩⎝ b a⎠ ⎭ ⎩⎝ 0 b⎠ ⎭
⎧⎛ a b c⎞ ⎫ ⎧⎛ x 0 x ⎞ ⎫
⎪⎜ ⎟ ⎪ ⎪⎜ ⎟ ⎪
9) ⎨⎜ 0 a b ⎟ | a, b, c ∈ Z⎬ 10) ⎨⎜ 0 0 0 ⎟ | x ∈ R ⎬
⎪⎜ 0 ⎪⎜ x 0 x ⎟ ⎪
⎩⎝ 0 a ⎟⎠ ⎪
⎭ ⎩⎝ ⎠ ⎭
⎧⎛ a 0 0 ⎞ ⎫
⎪⎜ ⎟ ⎪ ⎧⎛ a 0 ⎞ ⎫
11) ⎨⎜ 0 b 0 ⎟ | a, b, c ∈ R ⎬ 12) ⎨⎜⎜ ⎟⎟ | a ∈ Z ⎬
⎪⎜ 0 0 c ⎟ ⎪ ⎩⎝ 0 0 ⎠ ⎭
⎩⎝ ⎠ ⎭
⎧⎛ 2a 0 ⎞ ⎫
13) ⎨⎜⎜ ⎟⎟ | a ∈ Z ⎬
⎩⎝ 0 0 ⎠ ⎭
g) Arătaţi că fiecare din următoarele mulţimi de funcţii reale definite pe [− 1,1] , împreună cu
operaţiile obişnuite de adunare şi înmulţire, formează subinele unitare şi comutative ale inelului
funcţiilor definite pe [− 1,1] :
1) mulţimea funcţiilor continue
2) mulţimea funcţiilor pare
3) mulţimea funcţiilor polinomiale
4) mulţimea funcţiilor derivabile
5) mulţimea funcţiilor mărginite.
Rafinarea discuţiei de la inel la corp presupune automat ca inelul să fie unitar, în legătură cu
simetrizabilitatea oricărui element nenul în raport cu a doua lege de compoziţie.
Prezentăm mai detaliat două exemple speciale de corpuri.
1) Corpul cuaternionilor, introdus de R. Hamilton (1854), obţinut după introducerea a
două entităţi matematice j, k asemănătoare unităţii imaginare i, din C, pentru care prelungim
înmulţirea cu regulile: i 2 = j 2 = k 2 = −1, ij = − ji = k , jk = − kj = i, ik = − ki = j , reguli uşor de reţinut
dacă avem în vedere permutările circulare ale elementelor i, j, k în această ordine. Un cuaternion
este un număr de forma a + bi + cj + dk , unde a , b, c, d ∈ R . Pe mulţimea H a cuaternionilor, care
extinde C, definim adunarea şi înmulţirea elementelor
q1 = a1 + b1i + c1 j + d1k , q 2 = a 2 + b2 i + c2 j + d 2 k :
q1 + q 2 = a1 + a 2 + (b1 + b2 )i + (c1 + c 2 ) j + (d 1 + d 2 )k
q1 ⋅ q 2 = (a1a 2 − b1b2 − c1c 2 − d 1 d 2 ) + (a1b2 + a 2 b1 + c1d 2 − c 2 d 1 )i +
+ (a1c 2 − b1d 2 + a 2 c1 + b2 d 1 ) j + (a1 d 2 + b1c 2 − b2 c1 + a 2 d 1 )k
(în esenţă sunt regulile de calcul cu expresii algebrice suplimentate cu cele de mai sus).
E util să prezentăm şi extinderile noţiunilor de conjugat şi normă. Mai precis, pentru
q = a + bi + cj + dk ∈ H atunci conjugatul său este q = a − bi − cj − dk ∈ H şi funcţia normă:
N : H → R+ , N (q) = q ⋅ q = a 2 + b 2 + c 2 + d 2 .
E uşor de verificat că (H ,+,⋅) este inel unitar, cu elementele neutre:
0 = 0 + 0 ⋅ i + 0 ⋅ j + 0 ⋅ k ,1 = 1 + 0 ⋅ i + 0 ⋅ j + 0 ⋅ k . Pentru a arăta inversabilitatea oricărui

- 35 -
CERCUL DE MATEMATICA
q q
q ∈ H , q ≠ 0, q = a + bi + cj + dk (deci a 2 + b 2 + c 2 + d 2 > 0 ) uzităm de : q ⋅ = ⋅ q = 1 deci q
N ( q) N ( q )
q a b c d
este inversabil şi chiar putem explicita: q −1 = = − ⋅i − ⋅ j− ⋅ k cu q −1 ∈ H .
N (q) N (q) N (q) N (q) N (q )
Se verifică uşor faptul că acest corp nu este comutativ.
2) Dacă d ∈ N \ {1} este un întreg liber de pătrate (i.e. în descompunerea lui d în factori
primi nu apare niciun pătrat perfect), cu adunarea şi înmulţirea uzuale, mulţimea
[ ] {not
}
Q d = a + b d , a , b ∈ Q este un corp comutativ, numit corp pătratic, iar numerele de forma
a + b d , cu d întreg liber de pătrate şi a, b ∈ Q se numesc numere pătratice. Pentru numărul
[ ]
z = a + b d ∈ Q d definim previzibil conjugatul său pătratic: z = a − b d .
Raţiunea apariţiei unor astfel de corpuri e dată de rezolvarea, de exemplu, în Q 3 a [ ]
ecuaţiei x 2 = 3 pentru că în Q ea nu avea soluţii.
Considerăm utilă pentru elevii interesaţi de tema propusă şi prezentarea câtorva alte
exemple de subcorpuri, în raport cu operaţiile uzuale de adunare şi înmulţire:
{
1) a + b 3 , a, b ∈ Q } 2) {a + bi, a, b ∈ Q}

3) {a + b 3i, a, b ∈ Q}
⎧⎪ −1+ i 3 ⎫⎪
4) ⎨a + bε , ε = , a, b ∈ R ⎬
⎪⎩ 2 ⎪⎭

{
5) a + b 3 2 + c3 4 , a, b, c ∈ Q } ⎧⎛ a − b ⎞
6) ⎨⎜⎜ ⎟⎟ | a, b ∈ Q ⎬

⎩⎝ b a ⎠ ⎭
⎧⎛ a − 3b ⎞ ⎫ ⎧⎛ a + b 4b ⎞ ⎫
7) ⎨⎜⎜ ⎟⎟ | a, b ∈ Q ⎬ 8) ⎨⎜⎜ ⎟⎟ | a, b ∈ Q ⎬
⎩⎝ b a ⎠ ⎭ ⎩⎝ − b a − b ⎠ ⎭
⎧⎛ a − b − c − d ⎞ ⎫
⎪⎜ ⎟ ⎪
⎧⎛ a b ⎞ ⎫ ⎪⎜ b a − d c ⎟ ⎪
9) ⎨⎜⎜ ⎟⎟ | a, b ∈ Q ⎬ 10) ⎨⎜ ⎟ | a, b, c, d ∈ Q ⎬
⎩⎝ − b a ⎠ ⎭ ⎪⎜ c d a −b ⎪

⎪⎜⎝ d − c b a ⎟
⎠ ⎪
⎩ ⎭
Pentru final selectăm câteva din problemele înrudite cu tema anunţată, propuse la diverse
concursuri de matematică şi nu numai:
I) Fie α ∈ C \ Q cu proprietatea că mulţimea A = {a + bα , a , b ∈ Z} este inel faţă de
operaţiile uzuale din C. Ştiind că inelul are exact 4 elemente inversabile, să se arate că A = Z [i ] .
(Olimpiada Naţională – Suceava 1997)
II) Se consideră un inel A.
a) Să se arate că mulţimea Z( A) = {a ∈ A | ax = xa, pentru orice x ∈ A} este subinel al
inelului A.
b) Să se arate că dacă orice subinel comutativ al lui A este corp, atunci A este corp.
(Olimpiada Naţională – Vâlcea 2002)
III) Inelul întregilor pătratici. Fie d un număr întreg liber de pătrate; definim
[ ] {
Z d = x ∈ C | x = m + n d , m, n ∈ Z . Atunci: }
([ ] )
1) Z d ;+,⋅ este un subinel al corpului numerelor complexe, chiar domeniu de integritate.

2) Z [ d ] este izomorf
⎛m n⎞
cu inelul matricelor de forma ⎜⎜ ⎟⎟, m, n ∈ Z , în raport cu
⎝ dn m ⎠
operaţiile uzuale cu matrice.
[ ]
3) Inelele Z d şi Z d ' [ ]
sunt izomorfe dacă şi numai dacă d = d ' (un astfel de
izomorfism f între cele două inele invariază elementele lui Z şi atunci el este bine determinat de
( )
valoarea f d ).

-3636 -
CERCUL DE MATEMATICA SSM
[ ] { }
4) Subinelele unitare ale lui Z d sunt de forma An = a + bn d | a, b ∈ Z , n ∈ N H
[ ] ( )
5) Definim aplicaţia normă N : Z d → Z , N m + n d = m 2 − dn 2 . Dacă notăm cu
x = m − n d conjugatul întregului pătratic x = m + n d , se arată că N are proprietăţi asemănătoare
( [ ])
modulului: N ( x) = x ⋅ x, N ( xy) = N ( x) ⋅ N ( y ) . De aici: x ∈ U Z d ⇔ N ( x) ∈ U ( Z ) = {± 1}.
6) Grupul multiplicativ ale elementelor inversabile din Z [i ] este U (Z [i ]) = {± 1,±i}.
7) Dacă d ∈ {2,3,5}, atunci U (Z [d ]) conţine o infinitate de elemente şi putem găsi în U (Z [d ])
elemente pozitive oricât de mici (este suficient să găsim un singur element, considerând apoi
puterile acestuia şi conjugatele lor).
Problemele 1-6 sunt rezolvate în [3], iar 7 poate fi găsită în variantele examenului de
bacalaureat din ultimii ani.
IV) Legat de corpul numerelor pătratice mai inserăm şi :
([ ] )
1) Q d ;+,⋅ este subcorp al lui C (inversul elementului nenul a + b d este

2
1
a − db 2
( ) ( ) a
a − b d ∈ Q d , deoarece a 2 − db 2 ≠ 0 , altfel se ajunge la d = ± ∉ R \ Q ).
b
( )
2) Q d este izomorf cu mulţimea matricelor de forma ⎜⎜
⎛a b⎞
⎟⎟, a, b ∈ Q , care formează
⎝ db a ⎠
corp în raport cu operaţiile uzuale de adunare şi înmulţire.
( ) ( )
3) Corpurile Q d şi Q d ' sunt izomorfe dacă şi numai dacă d = d ' ; singurele
( )
automorfisme ale corpului Q d sunt aplicaţia identică şi cea de conjugare, ambele invariind
elementele lui Q.
4) Dacă un subcorp K ⊂ C este astfel încât EndK = { f , g} şi f ( x) = g ( x) ⇒ x ∈ Q , atunci
( )
există un întreg liber de pătrate d ≠ 1 pentru care K = Q d .
Olimpiada de Matematică Naţională, 1988 , Marcel Ţena
Problemele 1-3 pot fi găsite în [3].
Într-un viitor articol ne propunem să sintetizăm câteva elemente teoretice suplimentare
legate de aceeaşi tematică , esenţiale în fundamentarea ştiinţifică a rezolvării unor probleme mai
speciale de construcţii geometrice cu rigla şi compasul, de exemplu cele trei probleme celebre ale
antichităţii.

Bibliografie:
1.M.Ţena –Algebra.Structuri fundamentale pentru liceu , Ed. Corint ,Bucureşti , 1996
2.Gh. Ekcstein –Olimpiadele şi concursurile pentru clasele IX-XII , anii 2001-2008, Ed.
Bârchi , Timişoara
3.C. Niţă , T. Spircu –Probleme de structuri algebrice , Ed. Tehnică, Bucureşti , 1974
4.I. Tofan , C. Volf – Algebră-Inele , module , Teorie Galois , Ed. Matrix-Rom ,
Bucureşti , 2001
5.M . Ganga –Manual de clasa a XII a , Ed. Mathpress, Ploieşti , 2001
6.V. Schneider –Probleme de algebră pentru clasa a XII a , Ed. Valeriu , Craiova ,
1996

- 37 -
PROBLEME PROPUSE
Elevii vor rezolva probleme de la clasa pe care o urmează şi de la clasa inferioară.
Se pot rezolva şi problemele date la Ediţia a IV-a a concursului „Petre Sergescu” -
pagina 3. Soluţiile redactate pe foi format A5 se vor preda profesorului îndrumător.

IV.1. În 8 cutii se repartizează în mod egal 48 de bile roşii şi albastre. Care este
numărul minim de bile albastre care trebuie să existe, astfel încât oricum ar fi
repartizate bilele în cutii, în fiecare cutie să se găsească cel puţin o bilă albă?
Inst. Niculina Oprita
IV.2. Ionuţ îşi propune să rezolve un anumit număr de probleme. Dacă ar rezolva
cate 7 probleme pe zi le-ar termina într-un anumit număr de zile,însă el rezolvă
câte 10 probleme pe zi şi termină cu 3 zile mai devreme. Câte probleme a avut de
rezolvat Ionuţ?
Inst. Maria Ungureanu

V.1. Sa se afle numerele naturale x,y, z astfel incat : xy⋅z- 20090 = 23⋅251.
prof. Elena Rimnicianu
2009 − 1
10
V.2. Scrieti numarul ca suma de zece puteri cu aceeasi baza.
2008
prof. Elena Rimnicianu
5n + 3 5n + 3
V.3. Sa se arate ca oricare ar fi n∈N, numarul 3 +8 nu este patrat perfect.
prof. Elena Rimnicianu
V.4. Poate fi numarul x4k+10y+2 patrat perfect oricare ar fi x, y, k∈N?
prof. Elena Rimnicianu
V.5. Să se scrie numărul 5 • 3 2008 ca o diferenţă de două cuburi perfecte.
Prof. Nedeianu Dan
V.6. Să se rezolve în N ecuaţia: 2x+2y+2z = 4672, unde x<y<z
Prof. Ionică Constantin
V.7. Să se determine cifra x astfel ca numărul 76x08 să se dividă cu 31.
Prof. Ionică Constantin
V.8. Să se determine x, y∈ N ştiind că x2y + xy = 272.
Prof. Ionică Constantin
V.9. Dacă x,y sunt numere naturale cu x-y<2008, iar câtul şi restul împărţiri lui x
la y sunt 201, respectiv 8, să se afle numerele x si y.
Prof. Nedeianu Dan
V.10. Varstele tatalui, fiului si nepotului sunt exprimate prin numere prime, iar peste
5 ani varstele lor vor fi exprimate prin numere naturale patrate perfecte.
Cati ani are fiecare?
prof. Osain Victoria
V.11. Determinati numerele naturale x, y, z pentru care xy+yz+zx = 11 si x ≤ y ≤ z.
prof. Osain Victoria
V.12. Aflati numarul prim “p”, stiind ca p2-2, 2p2-1, 3p2 +4 sunt de asemenea prime.
prof. Osain Victoria

- 36 -
PROBLEME PROPUSE SSM
H

VI.1. Fie n∈N. Sa se arate ca numarul A = 22n (26n + 29n ) - 3n (5n+1) este divizibil
cu 101.
prof. Elena Rimnicianu
VI.2. Demonstrati ca daca n∈N si 11|4n +5 atunci 11|21n2+22n +5.
prof. Elena Rimnicianu
VI.3. Daca n∈N, n impar, a =2p, p∈N si k∈N, demonstrati ca fractia
a 3 k + a ⋅ n 2 k + a (n k + a ) + n
este ireductibila.
nk + a
prof. Elena Rimnicianu
VI.4. În triunghiul ABC cu m(Ĉ)=75 o , înalţimea [AD] este jumătate din latura [BC].
Să se afle m(Â).
Prof. Nedeianu Dan
1 1
VI.5. Aflaţi toate fracţiile ordinare cu numitorul 185 cuprinse între şi şi al căror
4 3
numărător este divizibil cu 7.
Prof. Ionică Constantin
VI.6. În ∆ ABC avem: m( A) = 4x + 100 şi m( B) = 6x - 600. Se cere:
a) Între ce limite poate varia x astfel ca ∆ ABC să existe?
b) Dacă ∆ ABC este dreptunghic să se determine x şi unghiurile ∆ ABC.
c) Să se afle valorile lui x pentru care ∆ ABC este isoscel.
Prof. Ionică Constantin
VI.7. Să se determine numerele naturale x, y, z distincte două cate două pentru care
xyz + xy + xz + yz + x + y + z = 2008.
Prof. Nedeianu Dan
VI.8. Intr-un patrulater convex triunghiurile formate de punctual de intersectie al
diagonalelor cu varfurile au acelasi perimetru. Sa se arate ca patrulaterul este
romb.
prof. Osain Victoria
VI.9. Sa se demonstreze ca din cifrele 1, 2, 3, 4, 5, 6, 7 nu se pot forma doua
numere de cate sapte cifre, diferite ( in fiecare numar fiecare cifra intra o singura
data) astfel incat unul sa se divida cu celalalt.
prof. Osain Victoria

VII.1. Să se calculeze valoarea numărului 10 2030 , unde indicii reprezintă baze de


40 50...
80 90

numeraţie. Evident, calculul se face de jos în sus.


Prof. Chirfot Carmen-Victoriţa
VII.2. Sa se afle numerele intregi x,y,z stiind ca : x2 –x + y − 3 + (z2 –9)2 ≤ 0.
prof. Elena Rimnicianu
VII.3. Triunghiul ABC are m(<ABC) = 2m(<ACB). Prin varful A ducem o paralela la
bisectoarea unghiului ABC, care intalneste pe BC in M. Daca (AB este bisectoarea
unghiului MAC stabiliti natura triunghiului ABC.
prof. Elena Rimnicianu

- 37 -
PROBLEME PROPUSE
2 1 1
VII.4. Rezolvati ecuatia + − = 2.
[ x ] {x} [ x]{x}
([x] si {x} reprezinta partea intreaga, respectiv partea fractionara a lui x).
prof. Elena Rimnicianu
VII.5. Sa se rezolve in R ecuatia 9{x}2-6x +1=0 , ({x} - partea fractionara a lui x).
prof. Elena Rimnicianu
VII.6. Determinati numerele reale a, b, c astfel incat
a+b+c = 3k si ab +ac +bc = 3k2, unde k ∈R.
prof. Elena Rimnicianu
VII.7. Ipotenuza BC a triunghiului dreptunghic ABC este împarţită de punctele M si N
AM + AN
în trei părţi egale. Calculaţi valoarea maximă a raportului .
BC
Prof. Nedeianu Dan
VII.8. Să se demonstreze că numărul 2005·2006·2007·2008 + 1 este pătrat perfect.
Prof. Ionică Constantin
VII.9. Să se determine n ∈ N pentru care n+2 + 3n − 5 = 7
Prof. Ionică Constantin
VII.10. În triunghiul ABC construim linia mijlocie DE, D∈ [AB], E∈ [AC] şi G
BG
centrul de greutate al triunghiului BCD, BG ∩ AC={F} . Calculaţi raportul .
GF
Prof. Nedeianu Dan
VII.11. Sa se arate ca ecuatia x2 +y2 =2007 nu are solutii numere intregi.
prof. Osain Victoria

VIII.1. Sa se arate ca 3⏐a3+b3 daca si numai daca 3 ⏐a+b.


prof. Elena Rimnicianu
⎡ x + 5⎤ ⎡ x + 6⎤ ⎡ x + 7⎤
VIII.2. Sa se rezolve in N ecuatia : ⎢ + + = 3 , unde [x] reprezinta
⎣ 6 ⎥⎦ ⎢⎣ 7 ⎥⎦ ⎢⎣ 5 ⎥⎦
partea inteaga a lui x.
prof. Elena Rimnicianu
VIII.3. Daca a, b, c ∈[0,1] cu ab + ac + bc = 1, demonstrati ca an +bn +cn ≤ 2, oricare
ar fi n∈N*.
prof. Elena Rimnicianu
VIII.4. Fie ABCD un pătrat de latura a si O punct oarecare în interior. Pe
a
perpendiculara in O pe planul (ABCD) se ia un punct V, cu OV= . Să se arate ca
2
suma pătratelor distanţelor de la V la laturile pătratului este mai mare sau egală
cu 2a 2 .
Prof. Nedeianu Dan
VIII.5. Ştiind că o prismă are 550 diagonale, să se determine numărul laturilor
poligonului convex de la baza ei.
Prof. Ionică Constantin
VIII.6. Să se arate că 2x + 6x + 9 +
2
2 x + y − 2 xy − 4 x + 4 ≥ 5,
2 2
x,y ∈ R.
Prof. Ionică Constantin

- 38 -
PROBLEME PROPUSE SSM
VIII.7. Să se rezolve în numere întregi ecuaţia: H
( x + 6) 2 + ( y + 2) 2 + ( x − 9) 2 + ( y − 3) 2 = 5 10
Prof. Ionică Constantin
a
VIII.8. Să se calculeze raportul , ştiind că a, b∈ R ∗ şi că a+2008 ab =2009b.
b
Prof. Nedeianu Dan
VIII.9. Sa se arate ca ecuatia 3x2 - 4xy + 2y2 - 21x + 12y – 3 = 0 nu are solutii intregi.
prof. Osain Victoria
VIII.10.Sa se arate ca ecuatia 2x +7y =19z nu are solutii numere naturale.
prof. Osain Victoria

IX.1. Dacă x2+y2+z2+t2+(x+y+z+t)2 = 5 , cu x,y,z,t ∈ R,


să se arate că x ≤ 2, y ≤ 2, z ≤ 2, t ≤ 2
Prof. Nedeianu Dan
⎧ x + [ y ] + {z} = 0,1

IX.2. Să se rezolve sistemul ⎨ y + [z ] + {x} = 1,2
⎪ z + [x ] + {y} = 1,3

Prof.Daniel Sitaru
IX.3. Sa se arate ca pentru orice numar natural n, exista un numar natural K avand
n cifre numai 1 si 2, cu proprietatea ca 2n divide pe K.
Prof. Cainiceanu George
⎧ x 2 + 3xy = 7
IX.4. Să se rezolve în mulţimea numerelor reale sistemul de ecuaţii: ⎪ 2
⎨ y − xy = 2
⎪x + y + z = 6

Prof. Grecu Vasile – Colegiul Economic
IX.5. Să se determine numerele reale strict pozitive care îndeplinesc condiţiile:
2 2 2 2 2 2
2x +y 2y +z 2z +x
≤ 1, ≤ 1, ≤1
xy + yz + zx xy + yz + zx xy + yz + zx
Prof. Grecu Vasile – Colegiul Economic
IX.6. Să se rezolve în R
x −1 x−2 x−3 x − 2006 x − 2005
+ + = x - 2007 + +
2007 2006 2005 2 3
prof. Daniel Stretcu – Colegiul Naţional “Gh. Ţiţeica”
IX.7. Dacă bn este o progresie geometrică cu termeni scrict pozitivi, să se arate că
bn ≥ b1 + (b2 − b1 )(n − 1), n ≥ 1.
Prof. Nedeianu Dan
IX.8. Să se determine cea mai mare valoare pentru S = x1x2 + x2x3 + … + +x2008x2009,
ştiind că x1, x2 … x2009 sunt numere nenegative cu proprietatea x1+ x2 + x3 + …
+ x2009 = 1
prof. Daniel Stretcu – Colegiul Naţional “Gh. Ţiţeica”
⎧ x − 1⎫ ⎧ x + 1 ⎫
IX.9. Rezolvati ⎨ ⎬=⎨ ⎬ (notam {x} partea fracţionară a nr. real x).
⎩ 2 ⎭ ⎩ 3 ⎭
prof. Daniel Stretcu – Colegiul Naţional “Gh. Ţiţeica”

- 39 -
PROBLEME PROPUSE
IX.10. Daca a,b ≥ 0, atunci [2a]+[2b]≥[a]+[b]+[a+b]
prof. Osain Victoria

X.1. Determinaţi z,y∈ Z , pentru care 2 ⋅ 3 x = 3 ⋅ 2 y + 246 .


Prof. Nedeianu Dan

2 2
X.2. Fie z1, z2∈C astfel încât z1 = 2009, z 2 = 2008 . Să se calculeze z1 − z 2 + z1 + z 2
Prof. Adi Lupu

X.3. Fie x . Consideram sirul


Determinati in functie de . Studiati cazul |x|
Prof. Giugiuc Constantin
⎧2 + 2 + y = a x −x
X.4. Sa se determine constanta a pentru care sistemul ⎨ 2
⎩ x + 2 = 3−a
y

admite o unica solutie reala.


Prof. Cainiceanu George
X.5. Fie n ∈ N , n > 2, z ∈ C , z ≠ 1 , cu proprietatea ca zn=1. Sa se arate ca:
2
a)|1-z|>
n −1
kπ 1
b) Pentru orice k ∈ Z nedivizibil cu n are loc inegalitatea | sin |>
n n −1
Prof. Cainiceanu George
1
X.6. a)Sa se arate ca functia f : R ∗ → R, f ( x) = x − nu este injective dar este surjectiva.
x
b)Se considera toate functiile injective pentru care exista m numar real nenul cu
proprietatea f ( x)( f o f )( x) = m, x ∈ R .Sa se arate ca f o f o f = f si sa se deduca apoi
forma generala a functiilor care verifica relatia data.
Prof. Cainiceanu George
X.7. a) Să se demonstreze că dacă a şi b sunt numere supraunitare, atunci:
a+b a+b
log a + log b ≥ 2.
2 2
b) Dacă x, y sunt numere reale strict pozitive, atunci:
( x + y) 2 ( x + y) 2
log x + y + log x + y ≥2
x
2 xy y
2 xy
Prof. Grecu Vasile – Colegiul Economic, Turnu Severin
X.8. Să se rezolve în mulţimea numerelor reale sistemul de ecuaţii:
⎧ lg x + lg y + lg z = 6 log x 10

⎨lg x + lg y + lg z = 12 log y 10
⎪lg x + lg y + lg z = 18 log 10
⎩ z
Prof. Grecu Vasile – Colegiul Economic, Turnu Severin
X.9. Rezolvaţi in C ecuaţia z 3 +3z 2 -3z+1=0.
Prof. Nedeianu Dan
k 1 1 1
X.10. Dacă z ∈ C , Re( z ) > , k ∈ R +* , arătaţi că − < .
2 z k k
prof. Daniel Stretcu – Colegiul Naţional “Gh. Ţiţeica”

- 40 -
PROBLEME PROPUSE SSM
X.11. Să se rezolve ecuaţia z3 = 5z + 47i , z = a + bi , cu a, b ∈ Z şi i2 = -1. H
prof. Daniel Stretcu – Colegiul Naţional “Gh. Ţiţeica”
X.12. Ai un cerc cu n puncte pe el. Fiecare 2 puncte distincte se unesc printr-un
segment. Se presupune ca aceste n puncte sunt distribuite pe cerc astfel incat nu
exista 3 segmente care sa se intersecteze in acelasi punct, in interiorul cercului
(cu alte cuvinte, numarul punctelor de intersectie intre aceste segmente, in
interiorul cercului, este maxim posibil). In cate sectiuni disjuncte este impartit
interiorul cercului de catre aceste segmente?
prof. Irina Zaman – Colegiul Naţional “Gh. Ţiţeica”

ab ab ab...
1424 3
ab ab

de n ori ab
XI.1. Calculaţi valoarea următoarei limite L = lim , unde indicii reprezintă
an n →∞

baze de numeraţie cu a ∈ {1, 2, ..., 9} si b ∈ {0, 1, 2,..., 9} .


Prof. Chirfot Carmen-Victoriţa
XI.2. Dacă A∈ M 3 (R) cu A2 = -2A , să se arate că det (A+2I3) ≥ det A.
Prof. Nedeianu Dan
[ x + 2 x + ln x]2
XI.3. Calculaţi lim , unde [x] partea întreagă a numărului real x.
x→∞ 5x + 2
Prof.Daniel Sitaru
⎛ 3 3 3⎞
⎜ ⎟
XI.4. Fie A = ⎜ 0 3 3 ⎟ şi B = A51 . Să se calculeze suma elementelor matricii B − T B.
⎜ 0 0 3⎟
⎝ ⎠
Prof.Daniel Sitaru
XI.5. Fie şirurile ( xn ) n ≥1 , ( y n ) n ≥1 , de numere raţionale care verifică relaţia:
x
(3 + 2 ) n = x n + y n 2 pentru orice n ≥ 1 . Se cere: lim n .
n →∞ y
n
Prof.Daniel Sitaru
x
⎛ 1− x 2− x 3− x 4− x

XI.6. Să se calculeze: lim ⎜⎜ 4 x
+4 x
+4 x
+4 x ⎟

x→∞
⎝ ⎠
Prof.Daniel Sitaru
XI.7. Fie A,B astfel incat exista Calculati
Prof.Grecu Vasile
XI.8. Fie A, B . Aratati ca daca
det|A+KB| este inversabila in
Prof.Trailescu Diana

XI.9. Fie f : [a, b ] → R , f functie ROLLE si derivabila pe [a,b]. Aratati ca exista o


vecinatate a lui a astfel incat f este crescatoare pe si o vecinatate a lui b
astfel incat f descrescatoare pe stiind ca
Prof. Giugiuc Constantin
XI.10. Fie . Determinati det in functie de tr(A) si det(A).
Prof.Giugiuc Leonard

- 41 -
PROBLEME PROPUSE
1 1 1
XI.11. Sa se calculeze lim (3 3 9 9 27 27 ....) ,daca produsul contine n factori.
n →∞
Prof. Cainiceanu George
XI.12. Considerăm a, b, c numere reale nenule , “ε” o rădăcină a ecuaţiei
⎛1 1 1 ⎞ ⎛a b c ⎞
⎜ ⎟ ⎜ ⎟
x + x + 1 = 0 şi matricele
2
E = ⎜1 ε ε ⎟ , F = ⎜ b c a ⎟ .
2

⎜1 ε 2 ε 4 ⎟ ⎜c a b⎟
⎝ ⎠ ⎝ ⎠
a) Să se arate că are loc egalitatea ε = −ε − 1 .2

b) Să se demonstreze că avem de asemenea ε 3 = 1 .


c) Să se demonstreze că det E = ε (ε + 1)(ε − 1) 3 .
d) Calculând det F după regula triunghiului sau regula lui Sarrus să se arate
că det F = 3abc − a 3 − b 3 − c 3 .
e) Calculând det F folosind proprietatea de adunare a liniei trei şi a liniei doi
peste linia unu, să se deducă formula
a 3 + b 3 + c 3 − 3abc = (a + b + c)(a 2 + b 2 + c 2 − ab − bc − ca) .
f) Dacă a, b, c sunt cifre şi d este cel mai mare divizor comun al numerelor
abc, bca, cab , să se arate că numărul det F este întreg şi se divide cu d.
g) Folosind formula det(E ∙F) =det E ∙ det F , să se arate că pentru
f ( x) = a + bx + cx 2 este adevărată egalitatea det F = − f (1) f (ε ) f (ε 2 ) .
Prof. Cainiceanu George
XI.13. Dacă a n este un şir de numere reale pozitive cu propietatea că
1
+ 3a n+1 < 4 , să se arate că şirul a n este convergent.
a3n
Prof. Nedeianu Dan

XII.1. Se consideră inelul (A, +, • ) inel cu cel puţin două elemente , cu propietatea că
xy+yz+zx=xyz, pentru orice x,y,z ∈ A-{0}. Să se arate că inelul este corp comutativ.
Prof. Nedeianu Dan
3 sin x + 2 cos x ⎛ π⎞
XII.2. Să se calculeze I=∫ dx; x ∈ ⎜ 0, ⎟
2 sin x + 3 cos x ⎝ 2⎠

J =∫
(
e x 6 xe x + 5e x + 2 x + 3 )
dx; x ∈ R .
x +1+ ex
Prof.Daniel Sitaru
XII.3. Se defineşte pe mulţimea R legea de compoziţie x*y=10xy+10x+10y+9.
a) Aflaţi a,b∈ Q–Z încât a*b∈ Z c) Aflaţi a,b∈ R–Q încât a*b∈ Z
b) Aflaţi a,b∈ Q–Z încât a*b∈ N d) Aflaţi a,b∈ R–Q încât a*b∈ N.
Prof.Daniel Sitaru
3
⎛ 4⎞
XII.4. Dacă f : R* → R; f ( x ) + 3 f ⎜ ⎟ = x ; x ∈ R * să se calculeze:
⎝x⎠
2
∫ f (x )dx .
1
Prof.Daniel Sitaru

- 42 -
PROBLEME PROPUSE SSM
π
2
H
XII.5. Fie a n = ∫ x n cos xdx . Să se calculeze: lim ( n 2 + n − n 2 + 1)a n .
n →∞
0
Prof.Daniel Sitaru
⎛ 2009 2008
2007 2007
2008 ⎞
2006
⎜ ⎟
XII.6. Să se arate că matricea A= ⎜ 20071007 2008 1008 1008
2009 ⎟ este inversabilă.
⎜ 3006 ⎟
⎝ 2008 2009 3008 2007 3007 ⎠
Prof.Adi Lupu

XII.7. Calculand , determinati suma .


Prof.Giugiuc Leonard
XII.8. În mulţimea M 2 ( Z 7 ) se consideră submulţimea M a tuturor matricilor
⎛ aˆ bˆ ⎞ ⎛ ⎞ ⎛ ⎞
M (aˆ, bˆ) = ⎜⎜ ⎟ , aˆ , bˆ ∈ Z 7 si matricile I 2 = ⎜ 1̂ 0̂ ⎟ , O2 = ⎜ 0̂ 0̂ ⎟ .
ˆ ⎟ ⎜ ⎟ ⎜ ⎟
⎝ b aˆ ⎠ ⎝ 0̂ 1̂ ⎠ ⎝ 0̂ 0̂ ⎠
a) Să se verifice că I 2 ∈ M şi O2 ∈ M .
b) Să se arate că {aˆ 3 aˆ ∈ Z 7 } = {0̂,1̂,6̂} .
c) Să se verifice că A ⋅ B ∈ M , A, B ∈ M .
d) Să se arate că ( M ,+,⋅) este inel.
e) Să se arate că funcţia f : M → Z 7 , f ( M ( aˆ , bˆ)) = aˆ + bˆ este morfism de inele.
f) Să se arate că ecuaţia x 3 + y 3 + z 3 = 2007 nu are soluţii numere întregi.
g) Să se rezolve ecuaţia X 6 = O2 , X ∈ M .
Prof. Manuela Prajea
1
XII.9. Se consideră a , b > 0 şi funcţia f : (0, ∞ ) → R, f ( x) = ( x + a )( + b ) .
x
a
a) Să se verifice că f ( x) = bx + + ab + 1 .
x
b) Să se calculeze lim f ( x) .
x→∞

c) Să se determine ecuaţia asimptotei oblice spre + ∞ a funcţiei f .


d) Să se calculeze f ′(x) .
e) Să se arate că f este convexă.
f) Să se arate că, dacă F : [c, d ] → R este funcţie convexă atunci maximul funcţiei F
este egal cu F (c) sau F (d ) .
g) Să se deduca inegalităţile:
1 1 1 9n 2
n 2 ≤ ( x1 + x2 + ... + x n )( + + ... + ) ≤ , n ∈ N * , x1 , x2 ,..., x n ∈ [1,2]
x1 x 2 xn 8
Prof. Manuela Prajea
XII.10. Se consideră funcţiile f : R → R , crescătoare pe (0, ∞) ,care au proprietatea:
f ( x + y ) + f ( x − y ) = 2( f ( x) + f ( y ) ) , x, y ∈ R si f (1) = 1 .
b) Să se verifice că funcţia g : R → R , g ( x) = x 2 satisface condiţiile din enunţ.
c) Să se arate că: f (− x) = f ( x) , x ∈ R .
d) Să se arate că f (rx ) = r 2 f ( x) , x ∈ R , x ∈ Q .
e) Să se dea exemplu de funcţie h : C → C care are proprietatea că

- 43 -
PROBLEME PROPUSE
h( z1 + z 2 ) + h( z1 − z 2 ) = 2(h( z1 ) + h( z 2 ) ) , z1 , z 2 ∈ C .
Prof. Manuela Prajea
x
XII.11. Se consideră funcţiile f ,g :[0 , ∝ )→R, f ( x) = ln(1 + x ) − şi
1+ x
g ( x) = ln(1 + x ) − x .
a) Să se calculeze f ' ( x) şi g ' ( x) .
b) Să se arate că f ( x) ≥ 0 , pentru orice x ≥0.
c) Să se arate că g ( x) ≤ 0 , pentru orice x ≥ 0.
1
d) Să se determine valoarea integralei definite ∫ f ( x)dx .
0

1
e) Să se demonstreze că 1 − ln 2 ≤ ln 4 − 1 ≤ .
2

f) Considerăm funcţia h( x) = ln(1 + x), h : [0,1] → R . Să se arate că dacă notăm


n
k n
n+k
s n = ∑ h( ) ,atunci avem s n = ln ∏ .
k =1 n k =1 n
1
1
g) Considerând cunoscut rezultatul că s n → ∫ h ( x)dx ,să se calculeze limita şirului (xn)
n 0

n (n + 1)(n + 2)...(n + n )
dat de formula xn = .
n
Prof. Cainiceanu George
π
XII.12. Calculati ∫ [sin x + cos x ] dx , ( notam [a]- partea întreagă a nr. real a).
0
Prof. Grecu Vasile – Colegiul Economic, Turnu Severin
a
XII.13. Calculati:
−a
∫ ( x + ln 1 − x 2 ) arcsin x dx , unde a ∈ (0, 1) .

Prof. Grecu Vasile – Colegiul Economic, Turnu Severin


3x 4 + 2 x 2 − 2 x − 1
XII.14. Calculaţi primitivele funcţiei f : (1, ∞ ) → R, f(x)=
x5 + x +1
Prof. Nedeianu Dan

- 44 -
PROBLEME PROPUSE SSM
OLIMPIADA DE MATEMATICĂ H
ETAPA LOCALĂ - 27 IANUARIE 2008
prof. Nanuti Dan
prof. Antonie Rodica
Clasa a V-a
I. Calculaţi: a)14 ⋅ 729 − 14 ⋅ 329 − 400 ;
2 2

[ ]
b) 3 7 ⋅ 310 + 2 201 : 2106 − 5 ⋅ (5 2 ) : (317 + 2 95 − 511 ) ;
5

c)1 + 2 − 3 + 4 + 5 − 6 + 7 + 8 − 9 + ... + 2005 + 2006 − 2007 + 2008 ;


Mariana Tatucu
II. În două lăzi se află un număr diferit de mere . Dacă se mută din lada mai plină în lada mai goală atâtea
mere câte sunt în lada goală, după trei operaţiuni de acest fel, în fiecare ladă rămân câte 240 de mere.
Câte mere au fost la început în fiecare ladă?
Rodica Antonie
III. Să se compare numerele a = card A şi b = card B , unde
{ } {
A = x x ∈ IN , 2 2007 p x ≤ 2 2008 şi B = y y ∈ IN , 1 ≤ y ≤ 31338 . }
Valeria Baloi
IV. Să se determine a, b ∈ IN astfel încât 6 a + 2 a +b ⋅ 3 a + 2 b = 17 ⋅ 37 − 1 .
Manuela Prajea
Clasa a-VI-a
I. Să se determine suma tuturor numerelor de forma abc , ştiind că ab , bc, ca sunt direct proporţionale
cu a + b, b + c, c + a .
II. Aflati cel mai mic număr natural care se scrie, în baza 10, numai cu cifrele 2 şi 3 şi se divide cu 132.
III. a)Se dau punctele A, B, C şi D , în această ordine, astfel încât 4 ⋅ AB + 5 ⋅ AD = 9 ⋅ AC şi BD = 18.
Să se afle lungimea segmentelor BC şi CD .
b)Fie semidreptele [OA, [OB, [OC şi [OD , în aceasta ordine, astfel încât m (∠AOD ) = 2 ⋅ m (∠BOC )
şi m(∠XOY ) = 90 o , unde [OX şi [OY sunt bisectoarele unghiurilor ∠AOB şi ∠COD . Să se
determine măsura unghiului BOC .
IV. Se consideră egalitatea : x 4 = 27 ⋅ y 4 + 2 ⋅ x 3 y , x, y ∈ IN .
a)Arătaţi că perechile x = 3 , y = 1 şi respectiv x = 9, y = 3 verifică egalitatea de mai sus.
b)Arătaţi că egalitatea din enunţ este verificată de o infinitate de perechi de numere naturale x şi y .
Dan Nedeianu
Clasa a-VII-a
I. Dacă 192abc este număr natural, să se arate că numărul (a + 1)(b + 2)c este pătrat perfect.
II. a)Câte soluţii întregi are inecuaţia : x ≤ 2008.
b) Câte soluţii naturale are ecuaţia: x + y + z = 2008.
III. Se consideră trapezul isoscel ABCD cu baza mare AB , m(∠ABD ) = 600 , AC ∩ BD = {O} ,
E mijlocul diagonalei [ AC ] şi F mijlocul diagonalei [BD] .
Dacă M este punctul egal depărtat de vârfurile A, B, C şi D ale trapezului, iar G este centrul de
greutate al triunghiului OEF , să se arate că G este mijlocul segmentului [MO ] .
Dan Nedeianu
IV. Se consideră dreptunghiul ABCD cu AB = a , AD = b şi a > b . Bisectoarea unghiului
BAD intersectează BD în E şi BC în F , iar paralela prin E la AB intersectează AC în G .
a)Determinaţi lungimea segmentului [EG ] .
b)Demonstraţi că FG ⊥ BD.
Selectate de Dana Paponiu si Carmen Coada

- 45 -
PROBLEME PROPUSE
Clasa a-VIII-a
a+b a− b
I. Dacă a > b >0 şi a 2 + b 2 = 10ab , calculaţi: şi .
a−b a+ b
Octavian Ungureanu
( )( )
II. Dacă n ∈ IN , n ≥ 2 şi x >0, să se arate că 1 + x + x + ... + x 1 + x − (2n + 2)x n ≥ 0
2 n n

Dan Nedeianu
III. Fie ABCD tetraedru cu propietăţile ( AB ) ≡ ( AC ) ≡ (BC ) ≡ ( AD ) ≡ (BD ) şi AD ⊥ BC .
Să se demonstreze că ABCD este tetraedru regulat.
Iuliana Gimoiu
IV. Se consideră triunghiul ABC dreptunghic cu m(∠A) = 90 o , AB = 6cm, AC = 8cm , iar punctele D
şi E sunt mijloacele segmentelor ( AB ) , respectiv ( AC ) . Pe prelungirea segmentului (DE ) se iau
punctele M şi N astfel încât triunghiurile MAB şi NAC să nu fie obtuzunghice , iar
MN = 12cm. În punctul M se ridică perpendiculara VM ⊥ ( ABC ) , VM = 1 cm .
a)Să se demonstreze că m(∠ANC ) = 90 o .
b)Să se calculeze distanţa de la punctul V la dreapta AB .
Dan Nedeianu
Clasa a IX-a
{ }
I. a.Fie A = ( x, y ) x + y ≤ 5 . Să se reprezinte grafic mulţimea A folosind ca unitate de măsură
“1cm”.
b.Să se arate că oricum am lua 101 puncte distincte din mulţimea A, există cel puţin două dintre
acestea aflate la o distanţă mai mică sau egală cu “1cm” unul de altul.
Daniel Sitaru
II. Fie a, b, c ∈ R variabile cu proprietatea că a + b + c = 18 . Să se determine valoarea maximă a
produsului a 2 b 3 c 4 şi să se determine a , b, c în acest caz.
George Cainiceanu
⎧ x − 1⎫ ⎧ x + 1⎫
III. Să se rezolve ecuaţia ⎨ ⎬=⎨ ⎬ , în mulţimea numerelor reale, unde {x} este partea
⎩ 2 ⎭ ⎩ 3 ⎭
fracţionară a numărului real x .
Daniel Sitaru
IV. Pe laturile [ AB], [ AC ] ale ∆ABC se consideră punctele D, respectiv E , astfel încât
DA + DB + EA + EC = 0 . Dacă O este mijlocul segmentului [DE ] şi AO I BC = {F } să se
BF
calculeze raportul .
FC
Dan Nedeianu
Clasa a X-a
27 1 1
I. Să se rezolve ecuaţia: 27 x + x
+ = x −1 + 3 x + 2 .
3 8 27
Dan Nedeianu
II. a)Fie x, y, z ∈ R . Să se demonstreze inegalităţile:
x 4 + y 4 + z 4 ≥ x 2 y 2 + y 2 z 2 + z 2 x 2 , x 4 + y 4 + z 4 ≥ xyz ( x + y + z ) .
b)Să se demonstreze că log 42 3 + log 42 5 + log 42 7 > log 2 7 ⋅ log 2 25 ⋅ log 2 27 .
Daniel Sitaru
2008
III. Fie z ∈ C astfel încât z ≤ 1 . Să se arate că 1 + ∑ (1 + i ) ⋅ z k < 2 2009 .
k

k =1
Ovidiu Ticusi
IV. Se consideră ecuaţia (E) x 28
− x − 1 = 0.
8

- 46 -
PROBLEME PROPUSE SSM
a)Dacă z este o rădăcină de modul 1 a ecuaţiei (E), să se arate că z 20
−1 = 1. H
b)Să se determine toate rădăcinile complexe de modul 1 ale ecuaţiei (E).
George Cainiceanu
Clasa a XI-a
⎛ 2008 − 2009 ⎞
I. Se dă matricea: A = ⎜⎜ ⎟⎟ .
⎝ 2007 − 2008 ⎠
a)Să se arate că A 2 − (TrA) ⋅ A + (det A) ⋅ I 2 = O2 , unde TrA este suma elementelor de pe
diagonala principală a matricei A.
b)Să se calculeze A 2008 şi A 2009 .
Adi Lupu
II. Se dă şirul de numere reale ( x n )n∈N pentru care x n+1 = 2007x n + 2008, n ∈ N si x0 = −1 .
a)Să se determine termenul general al şirului.
b)Să se studieze monotonia, mărginirea şi existenţa limitei şirului.
Doru Presneanu
III. Fie A ∈ M 3 (R ) . Ştiind că A = αA, unde α ∈ R − {1}, să se calculeze det A
t 2008
.
Adi Lupu, Doru Presneanu
(
IV. Calculaţi lim cos nπ 3 n 3 + 3n 2 + 6n + 5 .
n→ ∞
)
Dan Nedeianu
Clasa a-XII-a
I. Pe IR se consideră legea : x ∗ y = 3 xy + 6 x + 6 y + 10 .
a)Să se arate că : x ∗ y = 3( x + 2 )( y + 2 ) − 2 ;
x ∗4
b)Să se rezolve ecuaţia : 1 ∗2
x4 x ∗4 ∗3x = 3 4015 − 2 .
...4
2008 ori
Emilia Raducan
⎛ 1 1⎞ ⎛ 1 1⎞
II. Fie G1 = ⎜ − , ⎟ şi G2 = ⎜ − , ⎟ , unde a>0 şi b>0 şi a ≠ b .
⎝ a a⎠ ⎝ b b⎠
x+ y x+ y
Definim legile : x ∗ y = , x, y ∈ G1 şi x o y = , x, y ∈ G 2 .
1 + a xy
2
1 + b 2 xy
a)Arătaţi că (G1 ,∗) şi (G 2, o) sunt grupuri abeliene.
b)Arătaţi că (G1 ,∗) ≈ (G o) .
2,
Daniel Sitaru
cos 2006 x ⎡π π ⎤ ⎛π ⎞ ⎛π ⎞
III. Fie F ( x ) = 2007∫ 2008
dx , unde x ∈ ⎢ , ⎥ . Ştiind că F ⎜ ⎟ = 0 să se afle F ⎜ ⎟ .
sin x ⎣4 2⎦ ⎝4⎠ ⎝2⎠
Eleodor Popescu
⎡ π⎤
IV. a)Aratati că există a, b, c, d ∈ IR cu (1 + sin x + cos x ) = (a + b sin x )(c + d cos x ), x ∈ ⎢0, ⎥ .
2

⎣ 2⎦
1 + sin x + cos x ⎡ π⎤
b)Să se calculeze: ∫ dx , unde x ∈ ⎢0, ⎥ .
(1 + sin x )(1 + cos x ) ⎣ 2⎦
Dan Nedeianu
NOTA DIN PARTEA REDACTIEI
Ne cerem anticipat scuze colegilor care sunt autori ale unor probleme din concurs si nu sunt trecuti ca autori. Le solicitam
totodata sa ne anunte pentru a prezenta cuvenita erata.

- 47 -
PROBLEME PROPUSE
Test de verificare – admitere clasa a V-a
Colegiul National „Gheorghe Ţiţeica”
MATEMATICĂ
SUBIECTUL I
Aflaţi termenul necunoscut din:
[ 35 – 4 × ( 24 : 3 ) – ( a + 13 ) : 10 ] ×2 + 12 : 3 = 6
SUBIECTUL II
1) Suma dintre câtul şi restul unei împărţiri de numere naturale nenule este 30 . Restul este cu 2 mai mare
decât triplul câtului. Determinaţi deîmpărţitul ştiind că suma dintre acesta şi împărţitor este 2023.
2) Într-o urnă sunt 105 bile albe, negre şi roşii. Dacă extragem din urnă 81 de bile cel puţin una este albă,
dacă extragem 61 de bile cel puţin una este roşie şi dacă extragem 71 de bile cel puţin una este neagră.
Câte bile sunt de fiecare culoare?

LIMBA ROMÂNĂ
I Se dă textul:
„ Fantastic joacă rândunici zglobii
În cerul plin de umbră şi lumină,
Şi-i linişte adâncă în grădină,
Sub piersicii cu flori trandafirii …” (Şt. O. Iosif – Linişte )
Cerinţe:
1) Găsiţi câte un cuvânt cu înţeles asemănător pentru următorii termeni din text: fantastic, zglobii,
linişte, adâncă , trandafirii.
2) Despărţiţi în silabe cuvintele: zglobii, flori, rândunici, piersicii, umbră.
3) Transcrieţi din text două cuvinte cu înţeles opus şi precizaţi ce sunt ca părţi de vorbire
4) Precizaţi funcţia sintactică (parte de propoziţie) a cuvintelor: (în) cerul , -i , (cu ) flori, rândunici,
trandafirii.
5) Alcătuiţi o propoziţie după următoarea schemă:

S At P C C
subst. subst. vb. altă parte de vorbire subst.

II Realizaţi o compunere de 15 – 20 de rânduri în care personajul principal să fie o rândunică. Daţi un titlu
potrivit compunerii.
NOTĂ: Timp de lucru 2 ore. Toate subiectele sunt obligatorii. Se acordă 10p din oficiu.

Colegiul National „Gheorghe Ţiţeica”


MATEMATICĂ
SUBIECTUL I
Aflaţi termenul necunoscut din:
{ [ 5 × ( a + 21 ) – 55 × 4]:5 + 27:3}:10=4
SUBIECTUL II
1) Trei copii au împreună 2800 lei. Să se afle cât are fiecare copil ştiind că primul are cât ceilalţi doi
împreună, iar al doilea cu 100 lei mai mult decât al treilea.
2) Într-o urnă sunt 105 bile albe, negre şi roşii. Dacă extragem din urnă 81 de bile cel puţin una este albă,
dacă extragem 61 de bile cel puţin una este roşie şi dacă extragem 71 de bile cel puţin una este neagră.
Câte bile sunt de fiecare culoare?

- 48 -
PROBLEME PROPUSE SSM
LIMBA ROMÂNĂ H
I Se dă textul:
„Sub streşini cuiburile vechi
Ascund iar vesele perechi
De rândunele,
Şi cuiburi nouă se clădesc,
Iar eu, furat de-un vis, privesc
Zâmbind la ele …” (Şt. O. Iosif – Cântec)
Cerinţe:
6) Găsiţi câte un cuvânt cu înţeles asemănător pentru următorii termeni din text: vechi, (ascund) iar,
se clădesc, vesele , zâmbind.
7) Despărţiţi în silabe cuvintele: streşini, vechi, rândunele, cuiburile, perechi.
8) Precizaţi ce sunt ca parte de vorbire următoarele cuvinte din text: vechi , ascund , nouă, zâmbind,
ele.
9) Realizaţi schema completă a următoarei propoziţii (parte de propoziţie + parte de vorbire) :
„Sub streşini cuiburile vechi
Ascund iar vesele perechi
De rândunele …”
II Realizaţi o compunere de 15 – 20 de rânduri în care să descrieţi sosirea anotimpului preferat. Daţi un
titlu potrivit compunerii.
NOTĂ: Timp de lucru 2 ore. Toate subiectele sunt obligatorii. Se acordă 10p din oficiu.

Colegiul National „Traian”


14.06.2008
MATEMATICA
1) a) Efectuati [( 2 + 4 × 17 − 8 : 2) : 2 − 13] : 5 = 2p
b) Determinati numarul a din egalitatea 480 : (3 × a − 240) − 648 : 6 = 52 2p
2) Daca adunam triplul unui numar cu triplul altui numar , obtinem 150. Aflati numerele stiind ca dublul
diferentei lor este 40. 3p

3) a).Scrieti toate numerele naturale de doua cifre diferite care au suma cifrelor egala cu 4. 1p
b).Scrieti toate numerele naturale de cel mult trei cifre care au suma cifrelor egala cu 4. 1p

LIMBA SI LITERATURA ROMANA


Citeşte cu atenţie textul următor, pentru a răspunde cerinţelor:
Azi e prima zi de şcoală. Trei luni de vacanţă la ţară s-au spulberat ca un vis. De dimineaţă, mama
m-a dus la şcoală, ca să mă înscrie într-a treia primară; nu eram prea încântat să mă duc, căci gândul
zbura încă la ţară. Pe toate străzile, forfotă mare de copii; în cele două librării, o mulţime de taţi şi mame
se îngrămădeau să cumpere ghiozdane cu cureluşe, serviete şi caiete, iar în faţa şcolii se înghesuia atâta
lume! Omul de serviciu şi gardianul se străduiau din răsputeri să elibereze poarta.
( Edmondo de Amicis, Cuore, inimă de copil)
1. Când se petrece acţiunea din fragmentul citat? 0,5p
2. Unde şi-a petrecut copilul vacanţa de vară? 0,5p
3. Din ce cauză copilul nu era prea încântat că trebuie să meargă la şcoală? 0,5p
4. Găseşte cuvântul cu acelaşi înţeles, apoi cuvântul cu înţeles opus pentru: prima şi să cumpere. 1p
5. Alcătuieşte schema următoarei propoziţii: Trei luni de vacanţă la ţară s-au spulberat ca un vis 1,5p
6. Alcătuieşte două propoziţii în care substantivul vis să aibă funcţiile sintactice de subiect, respectiv, de
atribut. 1p
7. Imaginează-ţi că eşti personajul din textul dat. Continuă şirul întâmplărilor, folosind ca început
enunţul: Omul de serviciu şi gardianul se străduiau din răsputeri să elibereze poarta.
Compunerea ta să aibă 12 - 15 rânduri. Găseşte-i un titlu potrivit . 4p

- 49 -
PROBLEME PROPUSE
Colegiul National „Traian”
20.06.2002
MATEMATICA
1) Determina valoarea lui x daca 3 × 0 + 3 × 7 − {4 + [3 × 5 × (2 × 4 − 3 x) + 1 × 13] : 8} = 6 .
2) Daca adunam triplul unui numar cu triplul altui numar obtinem 165.Aflati numerele stiind ca triplul
diferentei numerelor este 45.
3) Suma a trei numere naturale este 6400. Daca se imparte primul numar la cel de-al doilea se obtine
catul 2 si restul egal cu cel de-al treilea numar. Aflati numerele stiind ca al treilea este mai mic cu 100
decat al doilea.

LIMBA SI LITERATURA ROMANA


1) Transforma vorbirea directa in vorbire indirecta:
“-Buna seara mos Martine!
-Buna vreme ,mai copile!
-Incotro cu noaptea-n cap?
-La oras sa-mi iau un brad.
-Pai in codru ,brain u ai?
-am,da-mi pare rau sa-I tai!”
2) Ilustreaza in enunturi proprii scrierea cuvintelor:
Nu mai | numai | nu m-ai; cai | ca-I; mie | mi-e; ai | a-i.
3) Analizeaza grammatical (sintactic si morfologic) cuvintele din versurile:
“Voi ati vantat cu glas fierbinte
Naturii calde imnuri sfinte.” (George Cosbuc—Vestitorii primaverii)

Colegiul National „Traian”


20.06.2001
MATEMATICA

1) Afla valoarea lui a din egalitatea [(20 + 339 : 3 + a : 4) × 7 − 5 × 400] : 3 − 22 × 3 + 125 × 8 = 1000
2) Suma a doua numere impartita la diferenta lor este 10 rest 1. Care sunt numerele, stiind ca unul este
mai mare decat celalalt cu 3.
3) Doi colegi citesc aceeasi carte. Dupa ce primul a citit 165 de pagini, iar cel de-al doilea 225 de pagini,
se constata ca primul mai are de citit de cinci ori mai mult decat al doilea. Cate pagini are cartea?

LIMBA SI LITERATURA ROMANA


1) “Un satean avea trei feciori care se tot certa. Din pricina aceasta el era tare suparat. Intr-o zi el ii
chema pe toti si le zise:
-care dintre voi ar putea sa franga acest manunchi de nuiele, capata o mie de lei.
Incercara pe rand ,dar n- au reusit.
-E peste putinta,zisera ei.
-Ba sa vedeti ca se poate. Batranul dezlega manunchiul si franse nuielele una cate una.
-Apoi asa stim si noi,zisera feciorii.
-Vedeti, asa e si cu voi. Daca veti fi uniti, veti fi puternici, daca nu, va poate frange oricine.
Printre feciori se lasa o tacere de mormant”
(“Tatal si feciorii”)
a) Povestiti textul, transformand vorbirea directa in vorbire indirecta.
b) Gasiti sinonime pentru cuvintele “manunchi”, “feciori”, “din pricina”, “sa franga”; alcatuiti cu
fiecare cate o propozitie.
C A P S A
c) Alcatuiti o propozitie dupa schema .
s. adj. v s p.

- 50 -
PROBLEME PROPUSE SSM
Colegiul National „Traian” H
16.06.2007
MATEMATICA
1) a)Sa se efectueze : [(91 − 4 × 19 + 15) : 6 − 2] × 4.
b)Sa se afle cate numere naturale de trei cifre au cifra sutelor egala cu dublul cifrei unitatilor, iar
cifra zecilor este mai mica decat cifra unitatilor.
2) Un sfert din lungimea unui teren dreptunghiular este cu 2m mai mare decat a treia parte din
latime. Sa se afle dimensiunile dreptunghiului stiind ca perimetrul sau este de 72m.

LIMBA SI LITERATURA ROMANA


1) Alcatuiti enunturi cu urmatoarele ortograme: va|v-a; mai|m-ai; car|c-ar; iau|i-au.
2) Se da textul:
Soarele scapata spre asfintit, departe, printer trestii. Inainte de fiorul amurgului,balta
tacea, solemna, oglindind in limpezimile ei un cer inalt, fara pata.
a) Indicati cuvinte cu acelasi inteles pentru: a scapata, asfintit, cer, a oglindi.
b) Analizati cuvintele subliniate din textul de mai sus.
3) Redactati o compunere de 15-20 de randuri, in care sa imbinati naratiunea si dialogul si care sa aiba
urmatorul sfarsit: …De-atunci nu m-am maid us niciodata cu tema nefacuta. Gasiti-I un titlu potrivit

- 51 -
Concursuri
OLIMPIADA NATIONALA DE MATEMATICA
2007 - 2008
Prencea Cassian V Medalia de bronz Paponiu Dana
Puican Tiberiu VI Medalie de aur,Mentiune MEDC Prajea Manuela
Stefan Andrei VII Medalie de bronz Prajea Manuela
Nistor Andreea VIII Medalie de bronz Ionica C-Tin
Carapencea Constantin X Medalie de bronz,Mentiune MEDC Paponiu Dana
Tigora Andrei XII Med.Bronz Cainiceanu Gh.

Olimpiada de matematicã - etapa judeţeană 01.03.2008 -


Clasa a V-a
Prencea Cassian CNT I Barzuica Alexandru Sc.nr.3 M
Verghelet Maria CNT II Botezatu Diana CNT M
Nicolicioiu Armand Titeica III Glonti Madalina CNT M
Pogacean Victor CNT III Grecu Andreea H CNT M
Nitoiu Mihai CNT M Rosoga Augusta Lic.Pedagogic M
Dirpes Iulian Menandu Sc.Voinescu M Amzoi Ileana CNT M
Cebuc Razvan CNT M Arnautu Catalina Sc.14 M
Prundeanu Sabina Sc.14 M Boruga Bogdan CNT M
Banica Tudor CNT M Chesa Maria Severinesti M
Chirita Dan CNT M Duican Irina Stefania Sc. nr. 14 M
Mihutescu Diana Sc.M.Viteazul M Dunarintu George Titeica M
Radoslav Teodor CNT M Lascu Robert CNT M
Ciuciu Simona CNT M Ochea Alin Titeica M
Daescu Maria Lic.Pedagogic M Pirvulescu Bianca Sc.P.Segescu M
Durus Andrei Sc.Voinescu M Robu Silviu Mihai Sc.14 M
Plavitu Victor CNT M Sacarin Claudiu CNT M
Stefanoiu Cristian CNT M Suta Alexandru CNT M
Andreescu Roxana CNT M Vaduva Beatrice G CNT M
Atomei Ciprian CNT M
Clasa a VI-a
Ionas Ioana Madalina Titeica I Baloi Elisabeta Sc. 2 M
Puican Tiberiu CNT II Popescu Corneliu CNT M
Ciuciu Laurentiu CNT III Iosu Raluca Sc. nr 6 M
Spinu Antonela Cristina Sc.Sergescu III Vladu Catalin Sc.Negreanu M
Ciobanu Diana CNT M Berbecariu Elena Sc.14 M
Ciuta Cora CNT M Borozia Daniel Cosmin Sc.Sergescu M
Troncota Diana CNT M Bosoanca Adriana D Sc.14 M
Benga Andrei CNT M Diaconescu Ovidiu G Sc.6 M
Gherasim Andrei CNT M Dragomir Andrei CNT M
Soare Daniela Ioana Sc.14 M Ene Bogdan Titeica M
Badescu Codrin CNT M Lupuleti Catalin Titeica M
Barbulescu Ramona CNT M Marghescu Luminita M Titeica M
Barbulescu Ramona Sc.14 M Petrescu Rucsandra V Sc.Sergescu M
Bojinovici Alexandra CNT M Prencea Vlad CNT M
Gheorghe Bogdan CNT M Semen Alexandra CNT M
Marzoca Cezara CNT M Visan Cosmina CNT M
Yassin Aylin CNT M

- 52 -
Concursuri SSM
Clasa a VII-a H
Numele şi prenumele Scoala Pr. Numele şi prenumele Scoala Pr.
Ştefan Andrei CNT I Driga Darie CNT M
Georgescu Ana CNT II Gavriloiu Costin Ţiţeica M
Tănasie Denisa CNT III Păcuraru Adrian Şc.Orşova M
Nicolae Andrei CNT M Păvălaşe Bogdan Ţiţeica M
Nuţă Flavius CNT M Băcanu George Şc. 6 M
Zamfirescu Simona CNT M Ciontea Ştefan Ţiţeica M
Chiriş Alexandra CNT M Ciucur Ştefan Ţiţeica M
Marghescu Andreea CNT M Gimoiu Ruxandra CNT M
Sbîrcea Alexandra CNT M Mitran Dragoş CNT M
Ţolea Cristian Ţiţeica M Voicu Răzvan CNT M
Boboiceanu Daniel Şc.14 M Arbănaşi Emil Ţiţeica M
Nistor Andreea Ţiţeica M Badea Beatrice CNT M
Săceanu Andrei CNT M Cătănescu Raluca Şc.Voinescu M
Ţirlui Valeria Ţiţeica M Diaconu Paula Lic.Odobleja M
Florescu Alexandru Ţiţeica M Dincă Diana Col.Decebal M
Anghel Cosmin Lic.V.Gomoiu M Gherghe Ana Maria Şc.Voinescu M
Anghel Cristian Lic.V.Gomoiu M Mituleţu Mădălina Ţiţeica M
Băcanu Alexandru CNT M Pristoleanu Narcis CNT M
Vâlcu Andrei CNT M Tripcea Roxana Şc.14 M
Aniţa Georgiana CNT M Zorilă Mihai CNT M
Borcean Diana Mădălina Şc. Severinesti M
Clasa a VIII-a
Numele şi prenumele Scoala Pr. Numele şi prenumele Scoala Pr.
Nistor Adriana Şc.14 I Cîrciu Bogdan Ţiţeica M
Nicolicioiu Andrei Ţiţeica II Constantinescu Robert Şc.Voinescu M
Prunescu Flavius CNT II Filipescu Alexandru Şc.14 M
Andreescu Mădălina CNT III Florichescu Alexandru Şc.Orşova M
Asproniu Robert CNT M Frăţilă Remus Şc.Voinescu M
Buţă Alexandra Simona Şc.Strehaia M Pănescu Dragoş CNT M
Toader Simona Şc.Voinescu M Vlaston Cristina Şc.6 M
Pacioagă Florentina Şc. Sergescu” M Zugravu Rozalia CNT M
Clasa a IX-a Clasa a X-a
Numele şi prenumele Scoala Pr. Numele şi prenumele Scoala Pr.
Teşilă Bianca CNT I Şeitan Mihaela CNT I
Papa Florin CNT II Carapencea Constantin CNT II
Popa Bogdan CNT III Pîrvulescu Dan CNT III
Văcărescu Mirela A Ţiţeica M Piţ Rada Andrei CNT M
Arcuşi Anamaria CNT M Calinovici Paul CNT M
Bălulescu Ligia CNT M Iorga Cristian Ţiţeica M
Radu Cornel Ţiţeica M Alexandru Crivac Cristina CNT M
Ştefan Radu Ţiţeica M Croitoru Răzvan CNT M
Copăceanu Adela Ţiţeica M Duţă Adrian CNT M
Curea Guţă Marius Ţiţeica M Marinescu Mihai CNT M
Doroiman Andreea Ţiţeica M Damian Beatrice CNT M
Dulan Ana CNT M Lolea Iulian CNT M
Fulga Iulia CNT M Ciotîrlă Dănuţ CNT M
Furcuţă Ioana CNT M Crăciunescu Marian CNT M
Glisca Marian CNT M Nicoară Călin CNT M
Milici Alina CNT M Petrescu Bianca CNT M
Pană Florin Claudiu CNT M Băltărete Ileana CNT M
Papa Mădălina CNT M Ciuciu Lucian Florin Ţiţeica M
Crîng Alisa Pedagogic M

- 53 -
Concursuri
Clasa a XI-a Clasa a XII-a
Numele şi prenumele Scoala Pr. Numele şi prenumele Scoala Pr.
Puţan Cătălin Ţiţeica I Tigora Andrei CNT I
Budirinca Alexandru CNT II Coandă Oana CNT II
Ciorobea Mihai CNT II Gogoloiu Gabriela Ţiţeica III
Dumbravă Roxana T. Lalescu III Mitucă Atena CNT M
Camalesa Bogdan Ţiţeica M Prundeanu Andreea CNT M
Dunăreanu Lidia Ţiţeica M Hinoveanu Cătălin CNT M
Mercioni Marina T. Lalescu M Popescu Anca Pedagogic M
Purcaru Alin Ţiţeica M Tuţă Leontin CNT M
Răducu Alexandru CNT M Răveanu Ioana CNT M
Şuşelea Robert CNT M
Botea Silvia CNT M
Ilie Mariana Pedagogic M
Lică Ştefania T. Lalescu M
Porojan Otilia CNT M

CONCURSUL DE MATEMATICĂ "ADOLF HAIMOVICI"


- etapa judeţeană 01.03.2008 -
Clasa a IX-a
Bran Adelina Ţiţeica I Ţîrlui Andreea T. Lalescu M
Dumitraşcu Roxana Economic II Alstani Cătălin D. Tudor M
Moraru Rebeca Pedagogic II Siclitaru Alexandra Economic M
Sitaru Daniela T. Lalescu II Sîbu Denisa Economic M
Vlădescu Mădălina Economic II Spătaru Alexandra Pedagogic M
Răescu Ionela Mihaela D. Tudor III Suta Alexandra CNT M
Sîrbu Denisa Ţiţeica III Ţogoe Luiza Economic M
Cunete Mihaela Decebal M
Clasa a X-a
Bobia Andreea Decebal I Dumitrana Manuela Economic II
Colţatu Elena Decebal I Ilie Liviu Costin Auto II
Avrămiuc Andrei Strehaia II Lupu Ana Strehaia II
Cazacu Laurnţiu Auto II Peagu Mihai Bogdan Auto II
Chiosa Răducu C-tin Auto II Şerban Bogdan D. Tudor II
Chiuescu Alina CNT II
Clasa a XI-a
Croitoru Bogdan Ţiţeica I Predescu George Ţiţeica M
Coliţă Georgiana Economic II Andriţa Caius Economic M
Anghel Nicolae Mihail Strehaia III Bădeţi Daniela Ţiţeica M
Prundeanu Marioara Economic III Giurescu Ştefania Ţiţeica M
Fica Andreea Ţiţeica III Iacobescu George D. Tudor M
Stoican Elena Economic III Strinoiu Eugen D. Tudor M
Vinjanu Ionel D. Tudor III Zamfirescu Daniel D. Tudor M
Bivolariu Ana Mari Strehaia M
Clasa a XII-a
Panfil Denisa Ţiţeica I Gheorghişan Paula Construcţii III
Atanasoaei Andrei Ţiţeica II Marian Andrei Vasile Ţiţeica M

Concursul national “Vranceanu-Procopiu”, Bacau,2007


Tesila Bianca IX mentiune prof. Paponiu Dana
prof. Carapencea Vali

- 54 -
Concursuri SSM
H
BARAJUL NATIONAL CANGURUL BUCURESTI
Tigora Andrei XII CNT PR.II
Carapencea Constantin X CNT PR.II
Sosu Cristian X CNT PR.II
Nicolicioiu Andrei VIII CNGT PR.III
Grosu Vlad VIII CNGT PR.III

BARAJUL JUDETEAN CANGURUL LA CLASELE 4-6


Popescu Cristiana IV Banica Teodor V
Raveanu Robert IV Cebuc Razvan V
Ion Miruna IV Verghelet Maria V
Benga Cristina IV Pogacean Victor V
Arbanasi Eliza IV Benga Andrei VI
Capastraru Bogdan IV Mema Serban VI
Butaru Nadina IV Ionas Ioana VI
Chicet Clara IV Puican Tiberiu VI
Chirita Dan V Troncota Diana VI
Prencea Cassian V Petrescu Lorena VI

Concursul National
de Evaluare in Matematica
CALIFICATI SI PREMIANTII LA ETAPA NATIONALA CONFORM SITE-ULUI FUNDATIEI
Tigora Andrei XII CNT premiul.II Voicu Razvan VII CNT
Puican Tiberiu VI CNT premiul.III Nistor Adriana VIII CNGT
Nicoara Calin X CNT premiul.III Nistor Mihaela VIII GEN.14
Nicolicioiu Armand V CNGT Radu Cornel IX CNGT
Ionas Ioana VI CNGT Stefan Radu IX CNGT
Lupuleti Catalin VI CNGT Carapencea C-Tin X CNT
Florescu Alexandru VII CNGT Stretcu Otilia X CNGT
Tanasie Denisa VII CNT Coanda Oana XII CNT
Tarlui Valeria VII CNGT

Concursul interjudetean “N.Coculescu”, Slatina, 2007


proba din 30.XI.2007 proba din 1.XII.2007
Pogacean Victor V mentiune Paponiu D Prencea Cassian V mentiune Paponiu D
Puican Tiberiu VI mentiune Prajea M Pogacean Victor V mentiune Paponiu D
Tesila Bianca IX premiul III Paponiu D Puican Tiberiu VI mentiune Prajea M
Carapencea C-tin X mentiune Paponiu D Tesila Bianca IX mentiune Paponiu D
Croitoru Razvan X mentiune Paponiu D Carapencea X mentiune Paponiu D
Constantin

Concursul interjudetean de matematica “Ion Ciolac”,


Craiova, 12.IV.2008
Prencea Cassian V mentiune prof. Paponiu Dana
Pogacean Victor V mentiune prof. Paponiu Dana
Puican Tiberiu VI premiul III prof. Prajea Manuela
Benga Andrei VI mentiune prof. Prajea Manuela
Stefan Andrei VII premiul I prof. Prajea Manuela
Zamfirescu Simona VII mentiune prof. Gimoiu Iuliana
Tesila Bianca IX premiul I prof. Paponiu Dana
Carapencea Constantin X mentiune prof. Paponiu Dana

- 55 -
Concursuri
Concursul national al revistei “Arhimede”
Etapa locala, 19.XI.2007, Craiova
Pogacean Victor V premiul II prof. Paponiu Dana
Prencea Cassian V premiul II prof. Paponiu Dana
Banica Teodor V premiul III prof. Paponiu Dana
Chirita Dan V premiul III prof. Paponiu Dana
Suta Adrian V premiul III prof. Paponiu Dana
Carapencea Constantin X mentiune prof. Paponiu Dana
Etapa zonala,Craiova , 23.II.2008
Pogacean Victor V premiul I prof. Paponiu Dana
Banica Teodor V premiul II prof. Paponiu Dana
Prencea Cassian V premiul II prof. Paponiu Dana
Carapencea Constantin X premiul II prof. Paponiu Dana
Etapa nationala, Craiova, 19.IV.2008
Pogacean Victor V premiul III
Banica Teodor V mentiune
Prencea Cassian V mentiune
Suta Adrian V mentiune
Carapencea Constantin X premiul II

Concursul interjudetean de matematica “Micul Arhimede”,Craiova


Pogacean Victor V mentiune prof. Paponiu Dana
Puican Tiberiu VI premiul I prof. Prajea Manuela
Ciuciu Laurentiu VI mentiune prof. Prajea Manuela
Benga Andrei mentiune prof. Prajea Manuela
Concursul national de matematica - “La scoala cu ceas”, Rm.Vilcea
proba de matematica
Verghelet Maria V premiul III si medalie de bronz prof. Paponiu Dana
Pogacean Victor V mentiune prof. Paponiu Dana
Prencea Cassian V mentiune prof. Paponiu Dana
Puican Tiberiu VI premiul III si medalie de bronz prof. Prajea Manuela
Zamfirescu Simona VII premiul III si medalie de bronz prof. Gimoiu Iuliana
Badea Beatrice VII premiul III si medalie de bronz prof. Prajea Manuela
Stefan Andrei VII mentiune prof. Prajea Manuela
Voicu Razvan VII mentiune prof. Prajea Manuela
Nicolae Andrei VII mentiune prof. Prajea Manuela
Prunescu Flavius VIII mentiune prof. Cainiceanu Gh.
Andreescu Madalina VIII mentiune prof. Antonie Rodica
proba tip ONM pentru clasele V-VI
Prencea Cassian V medalie de bronz prof. Paponiu Dana
Puican Tiberiu VI medalie de bronz prof. Prajea Manuela
Benga Andrei VI medalie de bronz prof. Prajea Manuela

Concursul Interjudetean Gheorghe Titeica 23.05.2008


Individual Probe pe echipaje
Verghelet Maria V CNT PR.I Prunescu Flavius VIII CNT PR.I
Prencea Cassian V CNT PR.II Stefan Andrei VII CNT PR.I
Pogacean Victor V CNT M Nistor Andreea VIII GEN.14 PR.I
Puican Tiberiu VI CNT PR.II Tesila Bianca IX CNT M
Prunescu Flavius VIII CNT M Seitan Mihaela X CNT M
Tesila Bianca IX CNT M Pirvulescu Dan X CNT M
Carapencea Constantin X CNT M Ciorobea Mihai XI CNT PR.II
Seitan Mihaela X CNT M Andreescu Madalina VIII CNT PR.III
Pirvulescu Dan X CNT M
Tigora Andrei XII CNT M

- 56 -
Concursuri SSM
H
Concursul Interjudeţean "Petre Sergescu" - 2008

clasa a IV-a
Burtea Cătălin Şc.Voinescu I Dragotescu Radu Andrei Şc. 6 III
Popescu Cristiana Şc.Paulian I Filip Radu Şc. 6 III
Paleacu Cosmin Şc.Voinescu II Lică Robert Şc.Voinescu III
Şontea Claudiu Şc.14 II Tănasie Dănuţ Şc.Voinescu III
Butaru Nadina Mădălina Şc.14 III Bălu Smaranda Şc.Voinescu III
Crăciun Madălina Şc.Voinescu III
clasa a V-a clasa a VI-a
Pogăcean Victor CNT I Puican Tiberiu CNT I
Calotă Dragoş CN.Carol II Trofin Raluca CN.Carol I
Chiriţa Dan CNT III Troncotă Diana CNT II
Benga Andrei CNT III
clasa a VII-a clasa a VIII-a
Ştefan Andrei CNT I Nistor Adriana Şc.14 I
Tănasie Denisa CNT II Băleanu Andrei Motru II
Anghel Cristian Lic.Gomoiu III Andreescu Mădălina CNT III
Gimoiu Ruxandra CNT III Semenescu Anca CD.Loga III
clasa a IX-a clasa a X-a
Teşilă Bianca CNT I Padureanu Victor CN Carol I
Miroşu Raluca CNT II Pirvulescu Dan CNT II
Agape Mihai CNT III Birovescu Georgiana CNT III
Alexandru Bogdan CN.Carol III
clasa a XI-a clasa a XII-a
Drăgoi Marius Motru I Stoian Bogdan Motru I
Butaru Nicu CNT II Tigora Andrei CNT I
Poşa Bogdan Motru II Goşea Ion Victor CN.Carol I
Ciorobea Mihai CNT III Mitucă Atena CNT II
Mema Alexandra CNT III Coandă Oana CNT III
Nistor Ovidiu CNT III Prundeanu Andreea CNT III

- 57 -
Concursuri
AMC 8 (noiembrie 2007)

Clasa a VI-a Clasa a VII-a


Puican Tiberiu CNT Gold Driga Darie Vlad CNT Gold
Mema Serban Costin CNT Silver Stuparu Denis CNT Silver
Ciobanu Diana CNT Bronze Barbuti Alexandru CNT Silver
Ciuciu Laurentiu CNT Bronze Zamfirescu Simona CNT Silver
Barbulescu Ramona CNT Bronze Marghescu Andreea CNT Bronze
Clasa a VIII-a Nicolae Andrei CNT Bronze
Asproniu Robert CNT Gold Saceanu Andrei CNT Bronze
Grosu Vlad CNT Gold Craciun Cosmin CNT Bronze
Nistor Andreea Şcoala Gen.14 Gold
Andreescu Madalina CNT Silver
Clain Andrei CNT Bronze
AMC 10/12 (februarie 2008)
Clasa a IX-a Clasa a X-a
Agape Mihai Locul I Sosu Cristian Locul I
Popa Andreea Locul II Nicoara Calin Locul II
Cepesi Cristian Locul III Calinovici Paul Locul III
Clasa a XI-a Clasa a XII-a
Nica Flavius Locul I Hinoveanu Catalin Locul I
Voicu Andreea Locul II Tigora Andrei Locul I
Bechir Adriana Locul III Steanta Anamarina Locul II
Nicolescu Andra Locul III Mariescu Radu Locul III
Papala Diana Locul III
AMERICAN INVITATION MATHEMATICAL EXAMINATION
Carapencea Constantin X CNT Coanda Oana XII CNT
Croitoru Razvan X CNT Mariescu Radu XII CNT
Tigora Andrei XII CNT Steanta Anamarina XII CNT
CIPHERING TIME TRIAL (Dec.2007)
Clasa a XII-a Clasa a X-a Clasa a XI-a Clasa a IX-a
Coanda Oana Calinovici Paul Voicu Andreea Papa Florin
Cainiceanu Andrei Sosu Cristian Nica Flavius Mirosu Raluca
Tigora Andrei Marin Andreca Cristina Mema Alexandra Agape Mihai
Hinoveanu Catalin Craciunescu Marian Daga Misu
Suselea Robert Furcuta Ioana

PURPLE COMET - APRILIE 2008


(UNIVERSITY OF WISCONSIN-WHITEWATER)

Mentiune - Echipaj clasa a VIII-a Mentiune - Echipaj liceu Mentiune - Echipaj liceu
Asproniu Robert Tigora Andrei Voicu Andreea
Andreescu Madalina Coanda Oana Hinoveanu Catalin
Grosu Vlad Carapencea Constantin Suselea Robert
Prunescu Flavius Seitan Mihaela Papa Florin
Rachitan Laura Croitoru Razvan Agape Mihai
Zugravu Rozalia Tesila Bianca Nicoara Calin

- 58 -
Concursuri SSM
Concursul “ First in math “ H
prof.Irina Zaman, Colegiul National”Gh. Titeica” Dr.Tr.Severin

In vara anului 2007, prof. Irina Zaman , de la C.N. “Gh. Titeica”, a


participat la Cursurile de vara ale Universitatii Rutgers din New Jersey, SUA,
obtinand o inscriere gratuita pt elevii sai, promotional, la Concursul American
“First in math”.
Elevii au primit un “user name” si o parola de acces pt
pag:www.firstinmath.com
Acest concurs s-a desfasurat on line, in anul scolar 2007-2008 si elevii
romani s-au clasat pe primele locuri, atat in topul liceelor cat si in top 100 –
clasament individual.

Acestea erau pozitiile elevilor romani pe 30 aprilie 2008 la finalul primului an:

19269 puncte Gheorghe Titeica, Drobeta Turnu Severin


in top 100 V-VIII-Alexandru Florescu

Gheorghe Titeica, Drobeta Turnu Severin - locul 4 in topul 100 al liceelor.

19269 Gheorghe Titeica, Drobeta Turnu Severin - in topul 100 al elevilor de cls
a VIII-a –Alexandru Florescu

Trofeul primit de Alex prin posta arata ca este pe primul loc in


Romania,pe locul 4 in lume la clasa sa si pe locul 20 in lume la clasele de
gimnaziu.

S-a facut o premiere in ian 2008 dupa ce s-au primit din SUA medalii ,
diplome,jocuri, bratari, etc si au obtinut:
Florescu Alexandru-aur
Fanea Alexandru-argint
Mituletu Madalina-bronz

Pentru cei pasionati de concursuri online, anul 2008 a inregistrat o premiera in


domeniu pentru Romania. Si anume, lansarea primului portal de concursuri
educationale online din tara noastra, care poate fi accesat la adresa
www.carepecare.ro. Startul a fost dat printr-un concurs de matematica cu
continut de nivelul clasei a III-a, dar care se adreseaza copiilor cu varste
cuprinse intre 8-12 ani. In cadrul aceluiasi portal urmeaza a fi lansate si alte
concursuri, care sa acopere toate grupele de varsta, dar si cat mai multe
discipline scolare. Cei mai buni participanti vor fi rasplatiti cu premii pe
masura. Mult succes la www.carepecare.ro!

- 59 -
REZOLVITORI

Colegiul National Traian


clasa a V-a Damsescu Iulia; clasa a VIII-a Mituca Anda; clasa a XI-a Budirinca Alexandru,
Buda Maria, Franculescu Diana, Cerga Alina, Zaharia Mihaela, Gridan Iulia, Ularu Ancuta,
Tanasescu Vlad, Vilcu Constantin, Mutu Marie-Jeane, Nistor Ovidiu, Nica Flavius,
Nicolescu Andra, Mema Alexandra, Picu Bianca, Stefanoiu Anca, Boboiceanu Silvia, Voicu
Andreea, Crivineanu Florin, Vulcanescu Adela, Enache Radu, Bolbotina Alexandru
clasa a XII-a Belbu Loredana, Porojan Otilia, Sandu Oana, Cotet Alexandra, Georgescu
Alin, Saftoiu Mihai, Modalca Denisa, Gamala Andreea, Trasca Mihai, Steanta Anamaria,
Paunescu Georgiana, Cristea Valentina, Buncianu Ilie, Ionica Adina, Marza Sebastian,
Stavaru Mihai, Mituca Anda, Botea Silvia, Ganda Irina, Badea Sabin, Nitulescu Maria, Cretu
Andrei, Tomescu Liana, Cucu Irina, Negrea Aurel, Raveanu Ioana, Mahut Alexandru, Pasat
Cristina, Paunescu Adrian, Cainiceanu Andrei, Iordache Norica, Vladu Margareta, Bojinovici
Sergiu, Hanes Anca

Scoala generală Nr.11


Prof. Victor Saceanu – Bestelei Ramona (10); Buica Ramona (10); Colta Ionut (10),
Covrescu Laurentiu (10); Cutitoiu Elvis (10); Dumitrascu Ramona (10); Gavrila Costinel
(10); Giubega Andreea (10); Gurgui Georgiana (10); Iancu Puiu Nicusor (10); Milotinov
Narcis (10); Mitroi Ionut (10); Pacala Georgiana (10); Radut Nicusor (10); Stanchescu Daniel
(10); Socate Georgiana (10).

Începând cu acest număr al revistei, se vor acorda patru premii, prin tragere
la sorţi, elevilor rezolvitori de probleme din revistă. Vor participa la tragerea
la sorţi toţi elevii care au predat profesorului de la clasă problemele rezolvate
(cel puţin una) şi talonul alăturat. Cele patru premii au valoarea de 50 lei.

Sponsorul concursului este S.C. ROCAST MEH EDI NŢI S.R.L.


reprezentată de dl. profesor Marcel Popescu

Tombola RMM
Numele şi prenumele elevului ____________________________________
Şcoala_______________________________________ clasa ___________
Numele şi prenumele profesorului ________________________________

Decembrie 2008
@ S.C. ROCAST MEHEDINŢI S.R.L.

- 60 -
COLABORATORI SSM
H
Gheorghe Cainiceanu Grecu Vasile
Trailescu Diana Giugiuc Leonard
Elena Rimnicianu Victor Săceanu
Osain Victoria Eleodor Popescu
Dan Nedeianu Daniel Stretcu
Ionică Constantin Gimoiu I uliana
Bondoc Gabriela Roxana Rodica Antonie
Draga Tătucu Mariana Paponiu Dana
Chilea I on Lupu Adrian
Manuela Prajea Raducan Emilia
Daniel Sitaru Ştefan Marica
Dan Nanuti Cristinel Mortici
Chirfot Carmen – Victoriţa I rina Zaman
Constantin Giugiuc elev. Nistor Adriana Mihaela
elev. I oana I onas

Colectivul de redacţie:
Gheorghe Căiniceanu
Prajea Manuela Nănuţi Dan
Dan Daniel Sitaru Dan
Antonie Rodica Stretcu Daniel
Ungureanu Octavian Nedeianu Dan
Popescu Eleodor Chilea Ion
Săceanu Victor Lupu Adrian
Decembrie 2008

- 61 -

S-ar putea să vă placă și